You are on page 1of 59

9.

MAGNETISM
THEORY
 
1. MAGNETIC FIELD AND FORCE If v and B are in the plane of paper, then according to

 Right-Hand Rule, the direction of F on positively charged
In order to define the magnetic field B , we deduce an expression particle will be perpendicular to the plane of paper upwards
for the force on a moving charge in a magnetic field. as shown in figure (a), and on negatively charged particle will
Consider a positive charge q moving in a uniform magnetic field be perpendicular to the plane of paper downwards, figure (b).
   
B , with a velocity V . Let the angle between V and B be .


(i) The magnitude of force F experienced by the moving charge
is directly proportional to the magnitude of the charge i.e.

Fq Definition of B
 If v = 1, q = 1 and sin  = 1 or = 90°, the nfrom (1),
(ii) The magnitude of force F is directly proportional to the
component of velocity acting perpendicular to the direction F = 1 × 1 × B × 1 = B.
of magnetic field, i.e. Thus the magnetic field induction at a point in the magnetic
F  v sin  field is equal to the force experienced by a unit charge moving
 with a unit velocity perpendicular to the direction of magnetic
(iii) The magnitude of force F is directly proportional to the field at that point.
magnitude of the magnetic field applied i.e.,
Special Cases
FB Case (i) If  = 0° or 180°, then sin = 0.
Combining the above factors, we get
 From (1),
F  qvsin B or F = kqv B sin  F = qv B (0) = 0.
where k is a constant of proportionality. Its value is found It means, a charged particle moving along or opposite to the
to be one i.e. k = 1. direction of magnetic field, does not experience any force.
 F = qv B sin  ...(1) Case (ii) If v = 0, then F = qv B sin = 0.
  

F  q vB  ...(2) It means, if a charged particle is at rest in a magnetic field, it
experiences no force.

The direction of F is the direction of cross-product of Case (iii) If = 90°, then sin = 1

velocity v and magnetic field B , which is perpendicular to  F = qv B (1) = qv B (Maximum).
 
the plane containing v and B . It is directed as given by the Unit of B . SI unit of B is tesla (T) or weber/(metre)2 i.e. (Wb/m2)
Right-handed-Screw Rule or Right-Hand Rule. or Ns C–1 m–1

Mahesh Tutorials Science


MAGNETISM 289

Thus, the magnetic field induction at a point is said to be speed, velocity, momentum and kinetic energy of charged
one tesla if a charge of one coulomb while moving at right particle will change.
angle to a magnetic field, with a velocity of 1 ms–1 experiences   
a force of 1 newton, at that point. Case II. When v, E and B are mutually perpendicular to
 
each other. In this situation if E and B are such that
MLT 2   
Dimensions of B    MA 1T 2 

AT LT 1  F  Fe  Fm  0 , then acceleration in the particle,

 F
a   0 . It means the particle will pass through the fields
2. LORENTZ FORCE m
without any change in its velocity. Here, Fe = Fm so qE = q v B
The force experienced by a charged particle moving in space or v = E/B.
where both electric and magnetic fields exist is called Lorentz This concept has been used in velocity-selector to get a
force. charged beam having a definite velocity.
Force due to electric field. When a charged particle carrying
 3. MOTION OF A CHARGED PARTICLE IN A
charge +q is subjected to an electric field of strength E , it
experiences a force given by UNIFORM MAGNETIC FIELD
 
Fe  qE ...(5) Suppose a particle of mass m and charge q, entering a


whose direction is the same as that of E . uniform magnetic field induction B at O, with velocity v ,
making an angle  with the direction of magnetic field acting
Force due to magnetic field. If the charged particle is moving
 in the plane of paper as shown in figure
in a magnetic field B , with a velocity v it experiences a
force given by
  

Fm  q v  B 
 
The direction of this force is in the direction of v  B i.e.

perpendicular to the plane contaning v and B and is
directed as given by Right hand screw rule.
Due to both the electric and magnetic fields, the total force
experienced by the charged particle will be given by
        
  
F  Fe  Fm  qE  q v  B  q E  v  B 
Resolving v into two rectangular components, we have :
    v cos  (= v1) acts in the direction of the magnetic field and

F  q E  vB  ...(6) v sin  (= v2) acts perpendicular to the direction of magnetic
field.
This is called Lorentz force.

Special cases For component velocity v2 , the force acting on the charged
   particle due to magnetic field is
Case I. When v, E and B , all the three are collinear.. In   
this situation, the charged particle is moving parallel or 
F  q v2  B 
antiparallel to the fields, the magnetic force on the charged
 
particle is zero. The electric force on the charged particle or F  q v 2  B  qv 2 Bsin 90  q  v sin   B ...(1)

 qE 
will produce acceleration a  , The direction of this force F is perpendicular to the plane
m  
containing B and v2 and is directed as given by Right
along the direction of electricl field. As a result of this, there
hand rule. As this force is to remain always perpendicular to
will be change in the speed of charged particle along the 
direction of the field. In this situation there will be no change v2 it does not perform any work and hence cannot change
in the direction of motion of the charged particle but, the 
the magnitude of velocity v2 . It changes only the direction

Lakshya Educare
290 MAGNETISM

of motion of the particle. Due to it, the charged particle is  


angle between v1 and B is zero. Thus the charged particle
made to move on a circular path in the magnetic field, as
shown in figure covers the linear distance in direction of the magnetic field
with a constant speed v cos .
Therefore, under the combined effect of the two component
velocities, the charged particle in magnetic field will cover
linear path as well as circular path i.e. the path of the charged
particle will be helical, whose axis is parallel to the direction
of magnetic field, figure

Here, magnetic field is shown perpendicular to the plane of


paper directed inwards and particle is moving in the plane
of paper. When the particle is at points A, C and D the
direction of magnetic force on the particle will be along AO,
CO and DO respectively, i.e., directed towards the centre O
of the circular path.
The force F on the charged particle due to magnetic field

2

provides the required centripetal force = mv 2 / r necessary
for motion along a circular path of radius r.

 Bq v 2  mv22 / r or v2  Bq r / m
or v sin  = B q r/m ...(2)
The angular velocity of rotation of the particle in magnetic
field will be

vsin  Bqr Bq
   The linear distance covered by the charged particle in the
r mr m magnetic field in time equal to one revolution of its circular
The frequency of rotation of the particle in magnetic field path (known as pitch of helix) will be
will be
2m
d  v1T  v cos 
 Bq Bq
v  ...(3)
2 2m Important points
The time period of revolution of the particle in the magnetic
1. If a charged particle having charge q is at rest in a magnetic
field will be 
field B , it experiences no force; as v = 0 and F = q v B sin  = 0.
1 2m 
T  ...(4)
v Bq 2. If charged particle is moving parallel to the direction of B , it
also does not experience any force because angle  between
From (3) and (4), we note that v and T do not depend upon  
velocity v of the particle. It means, all the charged particles
v and B is 0° or 180° and sin 0° = sin 180° = 0. Therefore,
the charged particle in this situation will continue moving
having the same specific charge (charge/mass) but moving
along the same path with the same velocity.
with different velocities at a point, will complete their circular
paths due to component velocities perpendicular to the 3. If charged particle is moving perpendicular to the direction
magnetic fields in the same time. 
of B , it experiences a maximum force which acts
 
For component velocity v1   vcos   , there will be no force perpendicular to the direction B as well as v . Hence this
on the charged particle in the magnetic field, because the force will provide the required centripetal force and the

Mahesh Tutorials Science


MAGNETISM 291

charged particle will describe a circular path in the magnetic sufficiently high energy with the help of smaller values of
oscillating electric field by making it to cross the same electric
mv 2
field of radius r, given by  Bqv . field time and again with the use of strong magnetic field.
r

4. MOTION IN COMBINED
ELECTRON AND MAGNETIC FIELDS

4.1 Velocity Filter


Velocity filter is an arrangement of cross electric and
magnetic fields in a region which helps us to select from a
beam, charged particles of the given velocity irrespective of
their charge and mass.
A velocity selector consists of two slits S1 and S2 held parallel
to each other, with common axis, some distance apart. In the
region between the slits, uniform electric and magnetic fields
are applied, perpendicular to each other as well as to the
axis of slits, as shown in figure. When a beam of charged
particles of different charges and masses after passing

through slit S1 enters the region of crossed electric field E

and magnetic field B , each particle experiences a force due
to these fields. Those particles which are moving with the Construction. It consists of two D-shaped hollow evacuated
velocity v, irrespective of their mass and charge, the force metal chambers D1 and D2 called the dees. These dees are
on each such particle due to electric field (qE) is equal and placed horizontally with their diametric edges parallel and
opposite to the force due to magnetic field (q v B), then slightly separated from each other. The dees are connected
q E = q v B or v = E/B to high frequency oscillator which can produce a potential
difference of the order of 104 volts at frequency  107 Hz.
The two dees are enclosed in an evacuated steel box and
are well insulated from it. The box is placed in a strong
magnetic field produced by two pole pieces of strong
electromagnets N, S. The magnetic field is perpendicular to
the plane of the dees. P is a place of ionic source or positively
charged particle figure.
Working and theory. The positive ion to be accelerated is
produced at P. Suppose, at that instant, D1 is at negative
potential and D2 is at positive potential. Therefore, the ion
will be accelerated towards D1. On reaching inside D1, the
Such particles will go undeviated and filtered out of the ion will be in a field free space. Hence it moves with a
region through the slit S2. Therefore, the particles emerging constant speed in D 1 say v. But due to perpendicular
from slit S2 will have the same velocity even though their magnetic field of strength B, the ion will describe a circular
charge and mass may be different.
The velocity filter is used in mass spectrograph which helps mv 2
path of radius r (say) in D1, given by Bqv  where m
to find the mass and specific charge (charge/mass) of the r
charged particle. and q are the mass and charge of the ion.
4.2 Cyclotron mv
 r
A cyclotron is a device developed by Lawrence and Bq
Livingstone by which the positively charged particles like Time taken by ion to describe a semicircular path is given
proton, deutron, alpha particle etc. can be accelerated.
r m 
Principle. The working of the cyclotron is based on the fact by, t    = a constant.
that a positively charged particle can be accelerated to a v Bq B  q / m 

Lakshya Educare
292 MAGNETISM

This time is independent of both the speed of the ion and in a conductor is due to motion of electrons, therefore,
radius of the circular path. In case the time during which electrons are moving from the end Q to P (along X’ axis).
the positive ion describes a semicircular path is equal to the
time during which half cycle of electric oscillator is completed,
then as the ion arrives in the gap between the two dees, the
polarity of the two dees is reversed i.e. D1 becomes positive
and D2 negative. Then, the positive ion is accelerated
towards D2 and it enters D2 with greater speed which remains
constant in D2. The ion will describe a semicircular path of
greater radius due to perpendicular magnetic field and again 
Let, vd drift velocity of electron
will arrive in a gap between the two dees exactly at the
instant, the polarity of the two dees is reversed. Thus, the – e = charge on each electron.
positive ion will go on accelerating every time it comes into Then magnetic Lorentz force on an electron is given by
the gap between the dees and will go on describing circular
  
path of greater and greater radius with greater and greater 
f   e vd  B 
speed and finally acquires a sufficiently high energy. The
accelerated ion can be removed out of the dees from window If n is the number density of free electrons i.e. number of
W, by applying the electric field across the deflecting plates free electrons per unit volume of the conductor, then total
E and F. number of free electrons in the conductor will be given by
Maximum Energy of positive ion N = n (A) = nA
Let v0, r0 = maximum velocity and maximum radius of the  Total force on the conductor is equal to the force acting on
circular path followed by the positive ion in cyclotron. all the free electrons inside the conductor while moving in
the magnetic field and is given by
mv02 Bqr0      
Then,
r0
 Bqv0 or v0 
m    
F  Nf  nA   e vd  B    nAe vd  B ...(7)
 

2
We know that current through a conductor is related with
1 1  Bqr0  B2q 2 r02 drift velocity by the relation
 Max. K.E.  mv02  m   
2 2  m  2m I = n A e vd
Cyclotron Frequency  I  nAevd .
If T is the time period of oscillating electric field then 
We represent I  as current element vector. It acts in the
T = 2t = 2 m/Bq  
direction of flow of current i.e. along OX. Since I  and vd
1 Bq have opposite directions, hence we can write
The cyclotron frequency is given by v  
T 2m  
I    nAevd ...(8)
It is also known as magnetic resonance frequency.
From (7) and (8), we have
The cyclotron angular frequency is given by   
F  I  B ...(9)
c  2v  Bq / m
  
F  I B
5. FORCE ON A CURRENT CARRYING CONDUCTOR
PLACED IN A MAGNETIC FIELD F  IBsin  ...(10)
 
were  is the smaller angle between I  and B .
Expression for the force acting on the conductor carrying
current placed in a magnetic field Special cases
Consider a straight cylindrical conductor PQ of length , Case I. If  = 0° or 180°, sin = 0,
area of cross-section A, carrying current I placed in a uniform From (10), F = IB (0) = 0 (Minimum)

magnetic field of induction, B . Let the conductor be placed It means a linear conductor carrying a current if placed parallel
along X-axis and magnetic field be acting in XY plane making to the direction of magnetic field, it experiences no force.
an angle  with X-axis. Suppose the current I flows through Case II. If  = 90°, sin = q ;
the conductor from the end P to Q, figure. Since the current
From (10), F = IB × 1 = IB (Maximum)

Mahesh Tutorials Science


MAGNETISM 293
  
It means a linear conductor carrying current if placed
perpendicular to the direction of magnetic field, it experiences The force on the arm QR is given by F2  I QR  B or  
maximum force. The direction of which can be given by F2 = I (QR) B sin  = I b B sin 
Right handed screw rule. The direction of this force is in the plane of the coil directed
downwards.
6. TORQUE ON A CURRENT CARRYING COIL IN  
A MAGNETIC FIELD Since the forces F2 and F4 are equal in magnitude and acting
in opposite directions along the same straight line, they cancel
Consider a rectangular coil PQRS suspended in a uniform out each other i.e. their resultant effect on the coil is zero.

magnetic field of induction B . Let PQ = RS =  and QR = SP = b. Now, the force on the arm PQ is given by
Let I be the current flowing through the coil in the direction     
PQRS and  be the angle which plane of the coil makes with   
F1  I PQ  B or F1 = I (PQ) B sin 90° = IB  PQ  B 
the direction of magnetic field figure. The forces will be
acting on the four arms of the coil. Direction of this force is perpendicular to the plane of the
coil directed outwards (i.e. perpendicular to the plane of
paper directed towards the reader).
And, force on the arm RS is given by
    
  
F3  I RS  B or F3 = I (PQ) B sin 90° = IB  RS  B 
The direction of this force, is perpendicular to the plane of paper
directed away from the reader i.e. into the plane of the coil.
The forces acting on the arms PQ and RS are equal, parallel
and acting in opposite directions having different lines of
action, form a couple, the effect of which is to rotate the coil
in the anticlockwise direction about the dotted line as axis.
The torque on the coil (equal to moment of couple) is given by
 = either force × arm of the couple
The forces F1 and F3 acting on the arms PQ and RS will be as
shown in figure when seen from the top.
Arm of couple = ST = PS cos  = b cos .
   IB  b cos   IBA cos  (  × b = A = area of coil
PQRS)
If the rectangular coil has n turns, then
  nIBA cos 
Note that if the normal drawn on the plane of the coil makes
an angle  with the direction of magnetic field, then +  = 90°
or = 90° – ; And cos = cos (90° – ) = sin 
Then torque becomes,
   
  nIBA sin   MBsin   M  B  nIA  B
   
Let F1 , F2 , F3 and F4 be the forces acting on the four current
where, nIA = M = magnitude of the magnetic dipole moment
carrying arms PQ, QR, RS and SP of the coil. of the rectangular current loop
The force on arm SP is given by,     
      M  B  nI A  B 
 
F4  I SP  B or F4 = I (SP) B sin (180° – ) = Ib B sin 
This torque tends to rotate the coil about its own axis. Its
 
 
The direction of this force is in the direction of SP  B i.e.
value changes with angle between plane of coil and direction
of magnetic field.
in the plane of coil directed upwards.

Lakshya Educare
294 MAGNETISM

Special cases 1. The lower end of the coil is connected to one end of a hair
If the coil is set with its plane parallel to the direction of spring S’ of quartz or phosphor bronze. The other end of this
magnetic field B, then highly elastic spring S’ is connected to a terminal T2. L is soft
iron core which may be spherical if the coil is circular and
  0 and cos   1 cylindrical, if the coil is rectangular. It is so held within the
 Torque,  = nIBA (1) = nIBA (Maximum) coil, that the coil can rotate freely without touching the iron
core and pole pieces. This makes the magnetic field linked
This is the case with a radial field.
with coil to be radial field i.e. the plane of the coil in all positions
2. If the coil is set with its plane perpendicular to the direction remains parallel to the direction of magnetic field. M is concave
of magentic field B, then  = 90° and cos  = 0 mirror attached to the phosphor bronze strip. This helps us to
 Torque, = nIBA (0) = 0 (Minimum) note the deflection of the coil using lamp and scale
arrangement. The whole arrangement is enclosed in a non-
7. MOVING COIL GALVANOMETER metallic case to avoid disturbance due to air etc. The case is
provided with levelling screws at the base.
Moving coil galvanometer is an instrument used for detection The spring S’ does three jobs for us : (i) It provides passage
and measurement of small electric currents. of current for the coil PQRS1 (ii) It keeps the coil in position
Principle. Its working is based on the fact that when a current and (iii) generates the restoring torque on the twisted coil.
carrying coil is placed in a magnetic field, it experiences a torque. The torsion head is connected to terminal T 1. The
Construction. It consists of a coil PQRS1 having large galvanometer can be connected to the circuit through
number of turns of insulated copper wire, figure. The coil is terminals T1 and T2.
wound over a non-magnetic metallic frame (usually brass) Theory. Suppose the coil PQRS1 is suspended freely in the
which may be rectangular or circular in shape. The coil is magnetic field.
suspended from a movable torsion head H by means of Let,  = length PQ or RS1 of the coil,
phosphor bronze strip in a uniform magnetic field produced
b = breadth QR or S1P of the coil,
by two strong cylindrical magnetic pole pieces N and S.
n = number of turns in the coil.
Area of each turn of the coil, A =  × b.
Let, B = strength of the magnetic field in which coil is
suspended.
I = current passing through the coil in the direction PQRS1
as shown in figure.
Let at any instant,  be the angle which the normal drawn on
the plane of the coil makes with the direction of magnetic field.
As already discussed, the rectangular coil carrying current
when placed in the magnetic field experiences a torque whose
magnitude is given by  = nIBA sin .
If the magnetic field is radial i.e. the plane of the coil is
parallel to the direction of the magnetic field then = 90°
and sin = 1.
 = nIBA
Due to this torque, the coil rotates. The phosphor bronze
strip gets twisted. As a result of it, a restoring torque comes
into play in the phosphor bronze strip, which would try to
restore the coil back to its original position.
Let  be the twist produced in the phosphor bronze strip
due to rotation of the coil and k be the restoring torque per
unit twist of the phosphor bronze strip, then total restoring
torque produced = k .
In equilibrium position of the coil, deflecting torque
= restoring torque

Mahesh Tutorials Science


MAGNETISM 295

 nIBA = k (b) The value of B can be increased by using a strong horse


shoe magnet.
k
or I  or I  G (c) The value of A can not be increased beyond a limit because
nBA
in that case the coil will not be in a uniform magnetic field.
k Moreover, it will make the galvanometer bulky and
where  G  a constant for a galvanometer. It is unmanageable.
nBA
known as galvanometer constant. (d) The value of k can be decreased. The value of k depends
upon the nature of the material used as suspension strip.
Hence, I   The value of k is very small for quartz or phosphor bronze.
It means, the deflection produced is proportional to the That is why, in sensitive galvanometer, quartz or phosphor
current flowing through the galvanometer. Such a bronze strip is used as a suspension strip.
galvanometer has a linear scale.
Current sensitivity of a galvanometer is defined as the 8. AMMETER
deflection produced in the galvanometer when a unit current
flows through it. An ammeter is a low resistance galvanometer. It is used to
measure the current in a circuit in amperes.
If  is the deflection in the galvanometer when current I is
passed through it, then A galvanometer can be converted into an ammeter by using
a low resistance wire in parallel with the galvanometer. The
Current sensitivity, resistance of this wire (called the shunt wire) depends upon
the range of the ammeter and can be calculated as follows :
 nBA  k 
Is    I   Let G = resistance of galvanometer, n = number of scale
I k  nBA 
divisions in the galvanometer,
The unit of current sensitivity is rad. A–1 or div. A–1. K = figure of merit or current for one scale deflection in the
Voltage sensitivity of a galvanometer is defined as the galvanometer.
deflection produced in the galvanometer when a unit voltage Then current which produces full scale deflection in the
is applied across the two terminals of the galvanometer. galvanometer, Ig = nK.
Let, V = voltage applied across the two terminals of the Let I be the maximum current to be measured by galvanometer.
galvanometer,
To do so, a shunt of resistance S is connected in parallel
 = deflection produced in the galvanometer. with the galvanometer so that out of the total current I, a
Then, voltage sensitivity, VS = /V part I g should pass through the galvanometer and the
If R = resistance of the galvanometer, I = current through it. remaining part (I – Ig) flows through the shunt figure
Then V = IR
 Voltage sensitivity,

 nBA IS
VS   
IR kR R
the unit of VS is rad V–1 or div. V–1.
Conditions for a sensitive galvanometer
A galvanometer is said to be very sensitive if it shows large
deflection even when a small current is passed through it.

nBA VA – VB = IgG = (I – Ig) S


From the theory of galvanometer,   I
k
 Ig 
For a given value of I,  will be large if nBA/k is large. It is so S G
or  I  Ig  ...(20)
if (a) n is large (b) B is large (c) A is large and (d) k is small.  
(a) The value of n can not be increased beyond a certain limit Thus S can be calculated.
because it results in an increase of the resistance of the If this value of shunt resistance S is connected in parallel
galvanometer and also makes the galvanometer bulky. This with galvanometer, it works as an ammeter for the range 0 to I
tends to decrease the sensitivity. Hence n can not be ampere. Now the same scale of the galvanometer which was
increased beyond a limit. recording the maximum current Ig before conversion into ammeter

Lakshya Educare
296 MAGNETISM

will record the maximum current I, after conversion into ammeter. V V


It means each division of the scale in ammeter will be showing From Ohm’s law, Ig  or G  R 
GR Ig
higher current than that of galvanometer.

V
or R G
Ig

Initial reading of each division of galvanometer to be used as If this value of R is connected in series with galvanometer, it
ammeter is Ig/n and the reading of the same each division works as a voltmeter of the range 0 to V volt. Now the same
after conversion into ammeter is I/n. scale of the galvanometer which was recording the maximum
The effective resistance R P of ammeter (i.e. shunted potential Ig G before conversion will record and potential V
galvanometer) will be after conversion in two voltmeter. It means each division of
the scale in voltmeter will show higher potential than that of
1 1 1 SG GS the galvanometer.
   or R P 
RP G S GS G S Effective resistance RS of converted galvanometer into
voltmeter is
As the shunt resistance is low, the combined resistance of
the galvanometer and the shunt is very low and hence RS = G + R
ammeter has a much lower resistance than galvanometer. An For voltmeter, a high resistance R is connected in series
ideal ammeter has zero resistance. with the galvanometer, therefore, the resistance of voltmeter
is very large as compared to that of galvanometer. The
resistance of an ideal voltmeter is infinity.
9. VOLTMETER
A voltmeter is a high resistance galvanometer. It is used to 10. BIOT-SAVART’S LAW
measure the potential difference between two points of a
circuit in volt. According to Biot-Savart’s law, the magnitude of the
magnetic field induction dB (also called magnetic flux
A galvanometer can be converted into a voltmeter by density) at a point P due to current element depends upon
connecting a high resistance in series with the galvanometer. the factors at stated below :
The value of the resistance depends upon the range of
voltmeter and can be calculated as follows : (i) dB  I (ii) dB  d
Let, G = resistance of galvanometer, 1
(iii) dB  sin  (iv) dB 
n = number of scale divisions in the galvanometer, r2
K = figure of merit of galvanometer i.e. current for one scale Combining these factors, we get
deflection of the galvanometer.
 Id sin 
Current which produces full scale deflection in the dB 
galvanometer, Ig = nK. r2
Let V be the potential difference to be measured by Id sin 
galvanometer. or dB  K
r2
To do so, a resistance R of such a value is connected in
series with the galvanometer so that if a potential difference
V is applied across the terminals A and B, a current Ig flows
through the galvanometer. figure

where K is a constant of proportionality. Its value depends


Now, total resistance of voltmeter = G + R on the system of units chosen for the measurement of the
various quantities and also on the medium between point P

Mahesh Tutorials Science


MAGNETISM 297

and the current element. When there is free space between 8. If  = 0° or 180°, then dB = 0 i.e. minimum.
current element and point, then Similarities and Dis-similarities between the Biot-Savart’s law
for the magnetic field and coulomb’s law for electrostatic field
0
In SI units, K  and In cgs system K = 1 Similarities
4
where 0 is absolute magnetic permeability of free space (i) Both the laws for fields are long range, since in both the
laws, the field at a point varies inversely as the square of the
and 0  4107 Wb A 1m1  4 107 TA 1m distance from the source to point of observation.
(ii) Both the fields obey superposition principle.
( 1 T = 1 Wb m–2)

(iii) The magnetic field is linear in the source Id  , just as the

In SI units, dB  0  Id sin  ...(3) electric field is linear in its source, the electric charge q.
4 r2

Id sin  11. MAGNETIC FIELD DUE TO A STRAIGHT


In cgs system, dB  CONDUCTOR CARRYING CURRENT
r2
In vector form, we may write Consider a straight wire conductor XY lying in the plane of
    paper carrying current I in the direction X to Y, figure. Let P
 0 I d   r
dB  or dB 

 0 I d   r ...(4)
be a point at a perpendicular distance a from the straight
4 r 3 4 r3 wire conductor. Clearly, PC = a. Let the conductor be made
of small current elements. Consider a small current element
   
Direction of dB . From (4), the direction of dB would Id  of the straight wire conductor at O. Let r be the
obviously be the direction of the cross product vector, position vector of P w.r.t. current element and  be the angle
  
d   r . It is represented by the Right handed screw rule or 
between Id  and r. Let CO = .

Right Hand Rule. Here dB is perpendicular to the plane

containing d  and r and is directed inwards. If the point P

is to the left of the current element, dB will be perpendicular

to the plane containing d  and r , directed outwards.
Some important features of Biot Savart’s law
1. Biot Savart’s law is valid for a symmetrical current distribution.
2. Biot Savart’s law is applicable only to very small length
conductor carrying current.
3. This law can not be easily verified experimentally as the
current carrying conductor of very small length can not be
obtained practically.
4. This law is analogous to Coulomb’s law in electrostatics.
 
5. The direction of dB is perpendicular to both Id  and r .
6. If  = 0° i.e. the point P lies on the axis of the linear conductor
carrying current (or on the wire carrying current) then 
According to Biot-Savart’s law, the magnetic field dB (i.e.
 0 Id sin 0 magnetic flux density or magnetic induction) at point P due
dB  0 
4 r2 to current element Id  is given by

 0 Id   r
It means there is no magnetic field induction at any point on
the thin linear current carrying conductor. dB  . 3
4 r
7. If  = 90° i.e. the point P lies at a perpendicular position w.r.t.
current element, then
 0 Id sin 
or dB   ...(5)
 Id 4 r2
dB  0 2 , which is maximum.
4 r

Lakshya Educare
298 MAGNETISM

In rt. angled POC, +  = 90° or = 90° – 


0 I  2I  2I L
 sin  = sin (90° – ) = cos  ...(6) Then, B  sin   sin   0 sin   40 a
4a 4 a 4a  L2
2

a a 
Also, cos   or r  ...(7) (iv) When point P lies on the wire conductor, then d  and r for
r cos 
each element of the straight wire conductor are parallel.

 Therefore, d   r  0 . So the magnetic field induction at P = 0.
And, tan   or   a tan 
a
Direction of magnetic field
Differentiating it, we get The magnetic field lines due to straight conductor carrying
d  a sec  d
2
...(8) current are in the form of concentric circles with the
conductor as centre, lying in a plane perpendicular to the
Putting the values in (5) from (6), (7) and (8), we get straight conductor. The direction of magnetic field lines is
anticlockwise, if the current flows from A to B in the straight

dB 
 

0 I a sec  d cos  0 I
2
cos  d ...(9)
conductor figure (a) and is clockwise if the current flows
4  a2  4 a from B to A in the straight conductor, figure (b). The direction
 2  of magnetic field lines is given by Right Hand Thumb Rule
 cos   or Maxwell’s cork screw rule.

The direction of dB , according to right hand thumb rule,
will be perpendicular to the plane of paper and directed
inwards. As all the current elements of the conductor will
also produce magnetic field in the same direction, therefore,
the total magnetic field at point P due to current through the
whole straight conductor XY can be obtained by integrating
Eq. (9) within the limits – 1 and + 2. Thus
2 2
 I  I
 cos  d  40 a sin 21

B  dB  0
4 a
1 1

0 I  I
 sin 2  sin  1    0  sin 1  sin 2  ...(10)
4 a 4 a Right hand thumb rule. According to this rule, if we imagine
the linear wire conductor to be held in the grip of the right
Special cases. (i) When the conductor XY is of infinite length
hand so that the thumb points in the direction of current,
and the point P lies near the centre of the conductor then
then the curvature of the fingers around the conductor
1  2  90 will represent the direction of magnetic field lines, figure
(a) and (b).
0 I  2I
So, B sin 90  sin 90  0 ...(11)
4 a 4 a
(ii) When the conductor XY is of infinite length but the point P
lies near the end Y (or X) then 1 = 90° and 2 = 0°.

0 I  I
So, B sin 90  sin 0  0 ...(11 a)
4 a 4 a
Thus we note that the magnetic field due to an infinite long
linear conductor carrying current near its centre is twice
than that near one of its ends.
(iii) If length of conductor is finite, say L and point P lies on
right bisector of conductor, then

L/2 L
1  2   and sin   
a 2   L / 2 4a  L2
2 2

Mahesh Tutorials Science


MAGNETISM 299

12. MAGNETIC FIELD AT THE CENTRE OF THE 0 I  2I


 B .2r  0
CIRCULAR COIL CARRYING CURRENT 4 r 2 4 r
If the circular coil consists of n turns, then
Consider a circular coil of radius r with centre O, lying with
its plane in the plane of paper. Let I be the current flowing in 0 2nI 0 I
B   2n ...(13)
the circular coil in the direction shown, figure (a). Suppose 4 r 4 r
the circular coil is made of a large number of current elements
each of length d. 0 I
i.e. B × angle subtended by coil at the centre.
4 r

Direction of B
The direction of magnetic field at the centre of circular current
loop is given by Right hand rule.
Right Hand rule. According to this rule, if we hold the thumb
of right hand mutually perpendicular to the grip of the fingers
such that the curvature of the fingers represent the direction
of current in the wire loop, then the thumb of the right hand
will point in the direction of magnetic field near the centre of
the current loop.

According to Biot-Savart’s law, the magnetic field at the



centre of the circular coil due to the current element Id  is
given by

 0  d   r 
dB  I 
4  r 3 

 0 Idr sin   0 Id sin 


or dB  
4 r3 4 r 2
where r is the position vector of point O from the current

element. Since the angle between d  and r is 90° (i.e.,  = 90°),
therefore, 13. AMPERE’S CIRCUITAL LAW
 0 Id sin 90  Id
dB  or dB  0 2 ...(12) Consider an open surface with a boundary C, and the current
4 r 2
4 r I is passing through the surface. Let the boundary C be
 made of large number of small line elements, each of length
In this case, the direction of dB is perpendicular to the 
plane of the current loop and is directed inwards. Since the d. The direction of d  of small line element under study is
current through all the elements of the circular coil will acting tangentially to its length d. Let Bt be the tangential
contribute to the magnetic feild in the same direction, component of the magnetic field induction at this element
therefore, the total magnetic field at point O due to current  
then Bt and d  are acting in the same direction, angle
in the whole circular coil can be obtained by integrating eq.
between them is zero. We take the product of Bt and d for
(12). Thus
that element. Then
 Id  0 I  

B  dB   40 
r 2 4 r 2
d  Bt d  B.d 

But  d = total length of the circular coil = circumference of the


current loop = 2r

Lakshya Educare
300 MAGNETISM

The relation (19) is independent of the size and shape of the


closed path or loop enclosing the current.

14. MAGNETIC FIELD DUE TO INFINITE LONG


STRAIGHT WIRE CARRYING CURRENT
Consider an infinite long straight wire lying in the plane of
paper. Let I be the current flowing through it from X to Y. A
magnetic field is produced which has the same magnitude
at all points that are at the same distance from the wire, i.e.
If length d is very small and products for all elements of
the magnetic field has cylindrical symmetry around the wire.
closed boundary are added together, then sum tends to be

an integral around the closed path or loop (i.e., ) . 


 
Therefore,  of B.d  over all elements on a closed path
  

 B.d  = Line integral of B around the closed path or
loop whose boundary coincides with the closed path.
According to Ampere’s circuital law,
 

B.d    0 I ...(19)

where I is the total current threading the closed path or loop


0 is the absolute permeability of the space. Thus, Let P be a point at a perpendicular distance r from the straight

a n d

Ampere’s circuital law states that the line integral of magnetic wire and B be the magnetic field at P. It will be acting
 tangentially to the magnetic field line passing through P.
field induction B around a closed path in vacuum is equal to
Consider an amperian loop as a circle of radius r, perpendicular
0 times the total current I threading the closed path. to the plane of paper with centre on wire such that point P
The relation (19) involves a sign convention, for the sense lies on the loop, figure. The magnitude of magnetic field is
of closed path to be traversed while taking the line integral 
same at all points on this loop. The magnetic field B at P
of magnetic field (i.e., direction of integration) and current
will be tangential to the circumference of the circular loop.
threading it, which is given by Right Hand Rule. According
We shall integrate the amperian path anticlockwise. Then
to it, if curvature of the fingers is perpendicular to the thumb  
of right hand such that the curvature of the fingers represents B and d  are acting in the same direction. The line integral
the sense, the boundary is traversed in the closed path or 
  of B around the closed loop is

loop for B.d  , then the direction of thumb gives the sense  
in which the current I is regarded as positive.   
B.d   Bd cos 0  B d  B2r

According to sign convention, for the closed path as shown As per sign convention, here I is positive,
in figure, I1 is positive and I2 is negative. Then, according to Using Ampere’s circuital law
Ampere’s circuital law  
 
B.d   0  I1  I 2    0 Ie
B.d    0 I or B2r   0 I

0 I 0 2I
where Ie is the total current enclosed by the loop or closed path. or B  ...(21)
2r 4 r

15. MAGNETIC FIELD DUE TO CURRENT THROUGH


A VERY LONG CIRCULAR CYLINDER
Consider an infinite long cylinder of radius R with axis XY.
Let I be the current passing through the cylinder. A magnetic
field is set up due to current through the cylinder in the form
of circular magnetic lines of force, with their centres lying

Mahesh Tutorials Science


MAGNETISM 301

on the axis of cylinder. These lines of force are perpendicular


 0 r Ir
to the length of cylinder. or B i.e., B  r
2R 2
If we plot a graph between magnetic field induction B and
distance from the axis of cylinder for a current flowing through
a solid cylinder, we get a curve of the type as shown figure

Case I. Point P is lying outside the cylinder. Let r be the


perpendicular distance of point P from the axis of cylinder, Here we note that the magnetic field induction is maximum
 for a point on the surface of solid cylinder carrying current
where r > R. Let B be the magnetic field induction at P. It is
acting tangential to the magnetic line of force at P directed and is zero for a point on the axis of cylinder.
 
into the paper. Here B and d  are acting in the same direction.
16. FORCE BETWEEN TWO PARALLEL CONDUCTORS
Applying Ampere circuital law we have CARRYING CURRENT
 
B.d   0 I or 
Bd cos 0   0 I Consider C 1D 1 and C 2 D 2, two infinite long straight
conductors carrying currents I1 and I2 in the same direction.
or  Bd  0I or B2r   0 I They are held parallel to each other at a distance r apart, in
the plane of paper. The magnetic field is produced due to
current through each conductor shown separately in figure.
0 I
or B , i.e., B  1/ r Since each conductor is in the magnetic field produced by
2r the other, therefore, each conductor experiences a force.
Case II. Point P is lying inside cylinder. Here r < R. we may
have two possibilities. D1 r D2
(i) If the current is only along the surface of cylinder which is B1 90° I2
so if the conductor is a cylindrical sheet of metal, then current
through the closed path L is zero. Using Ampere circutal F1 F2
B
law, we have B = 0. × ×
(ii) If the current is uniformly distributed throughout the cross- B
section of the conductor, then the current through closed 90°
path L is given by
I1 B2
2
I Ir C1 C2
I'   r 2  2
R 2
R
Magnetic field induction at a point P on conductor C2D2
Applying Ampere’s circuital law, we have
due to current I1 passing through C1D1 is given by
 
B.d    0 r I '
B1 
0 2I1
...(12)
4 r
0 r Ir 2
or 2rB  0 r I '  According to right hand rule, the direction of magnetic field
R2 
B1 is perpendicular to the plane of paper, directed inwards.

Lakshya Educare
302 MAGNETISM

As the current carrying conductor C2D2 lies in the magnetic


   Q  R  S  P 
field B1 (produced by the current through C1D1), therefore,     
B.d   B.d   B.d   B.d   B.d 
PQRS P Q R S
the unit length of C2D2 will experience a force given by
F2 = B1I2 × 1 = B1I2 Q
  Q
Putting the value of B1, we have  
Here, B.d   Bd cos 0  BL
P P
 2I I
F2  0 . 1 2 ...(13)
4 r R
  R P
 
It means the two linear parallel conductors carrying
and   
B.d   Bd cos90  0  B.d 
Q Q S
currents in the same direction attract each other.
Thus one ampere is that much current which when flowing S
 
through each of the two parallel uniform long linear
conductors placed in free space at a distance of one metre

Also, B.d   0 ( outside the solenoid, B = 0)
R
from each other will attract or repel each other with a force
 
of 2 × 10–7 N per metre of their length.
 B.d   BL  0  0  0  BL
...(21)
PQRS
17. THE SOLENOID
From Ampere’s circuital law
A solenoid consists of an insulating long wire closely wound  
in the form of a helix. Its length is very large as compared to  B.d   0 × total current through the rectangle PQRS
its diameter. PQRS

Magnetic field due to a solenoid = 0 × no. of turns in rectangle × current


Consider a long straight solenoid of circular cross-section. = 0 n LI ...(22)
Each two turns of the solenoid are insulated from each other. From (21) and (22), we have
When current is passed through the solenoid, then each
turn of the solenoid can be regarded as a circular loop BL = 0 n LI or B = 0 n I
carrying current and thus will be producing a magnetic field. This relation gives the magnetic field induction at a point
At a point outside the solenoid, the magnetic fields due to well inside the solenoid. At a point near the end of a solenoid,
neighbouring loops oppose each other and at a point inside the magnetic field induction is found to be 0 n I/2.
the solenoid, the magnetic fields are in the same direction.
As a result of it, the effective magnetic field outside the 18. TOROID
solenoid becomes weak, whereas the magnetic field in the
interior of solenoid becomes strong and uniform, acting The toroid is a hollow circular ring on which a large number of
along the axis of the solenoid. insulated turns of a metallic wire are closely wound. In fact, a
toroid is an endless solenoid in the form of a ring, figure.
Let us now apply Ampere’s circuital law.
Let n be the number of turns per unit length of solenoid and
I be the current flowing through the solenoid and the turns
of the solenoid be closely packed.
Consider a rectangular amperian loop PQRS near the middle
of solenoid as shown in figure

S R

P Q
L
B

××××××××××××××××× Magnetic field due to current in ideal toroid


 Let n be the number of turns per unit length of toroid and I
The line integral of magnetic field induction B over the be the current flowing through it. In case of ideal toroid, the
closed path PQRS is coil turns are circular and closely wound. A magnetic field

Mahesh Tutorials Science


MAGNETISM 303

of constant magnitude is set up inside the turns of toroid in


the form of concentric circular magnetic field lines. The 19. MAGNETISM & MATTER
direction of the magnetic field at a point is given by the
tangent to the magnetic field line at that point. We draw 19.1 The Bar Magnet
three circular amperian loops, 1, 2 and 3 of radii r1, r2 and r3 to
be traversed in clockwise direction as shown by dashed It is the most commonly used form of an artificial magnet.
circles in figure, so that the points P, S and Q may lie on
When we hold a sheet of glass over a short bar magnet and
them. The circular area bounded by loops 2 and 3, both cut
sprinkle some iron filings on the sheet, the iron filings
the toroid. Each turn of current carrying wire is cut once by
rearrange themselves as shown in figure. The pattern
the loop 2 and twice by the loop 3. Let B1 be the magnitude
suggests that attraction is maximum at the two ends of the
of magnetic field along loop 1. Line integral of magnetic
bar magnet. These ends are called poles of the magnet.
field B1 along the loop 1 is
 
 B1 .d    B1d cos 0  B1 2r1 ...(i)
loop 1 loop 1

Loop 1 encloses no current.


According to Ampere’s circuital law
 
 B1 .d   0  current enclosed by loop 1 = 0 × 0 = 0
loop 1

or B12  r1 = 0 or B1 = 0
Let B3 be the magnitude of magnetic field along the loop 3.
The line integral of magnetic field B3 along the loop 3 is
 
 B3 .d    B3d cos 0  B3 2r3
loop 3 loop 3

From the sectional cut as shown in figure, we note that the


current coming out of the plane of paper is cancelled exactly
by the current going into it. Therefore, the total current
enclosed by loop 3 is zero.
According to Ampere’s circuital law
 
 B3 .d   0 × total current through loop 3
loop 3

or B3 2r3  0  0  0 or B3  0
Let B the magnitude of magnetic field along the loop 2. Line
integral of magnetic field along the loop 2 is 1. The earth behaves as a magnet.
 
 B.d   B2r2 2. Every magnet attracts small pieces of magnetic substances
like iron, cobalt, nickel and steel towards it.
loop 2
3. When a magnet is suspended freely with the help of an
Current enclosed by the loop 2 = number of turns × current unspun thread, it comes to rest along the north south
in each turn = 2  r2 n × I direction.
According to Ampere’s circuital law 4. Like poles repel each other and unlike poles attract each
  other.
 B.d   0  total current
5. The force of attraction or repulsion F between two magnetic
loop 2
poles of strengths m1 and m2 separated by a distance r is
or B2  r2   0  2r2 nI or B   0 nI directly proportional to the product of pole strengths and
inversely proportional to the square of the distance between
their centres, i.e.,

Lakshya Educare
304 MAGNETISM

m1m 2 mm the field of the magnet. The torque acting on a compass


F 2
or F  K 1 2 2 , where K is magnetic force needle aligns it in the direction of the magnetic field.
r r
constant. The path along which the compass needles are aligned is
known as magnetic field line.
0
In SI units, K   107 Wb A 1m 1
4
where 0 is absolute magnetic permeability of free space
(air/vacuum).

0 m1m 2
 F ...(1)
4 r 2
This is called Coulomb’s law of magnetic force. However, in
cgs system, the value of K = 1.

This corresponds to Coulomb’s law in electrostatics.

SI Unit of magnetic pole strength


Suppose m1 = m2 = m (say),
r = 1 m and F = 10–7 N
From equation (1),

107  107 
m m or m  1 or m = +1 ampere-metre
2
12
(Am). Therefore, strength of a magnetic pole is said to be
one ampere-metre, if it repels an equal and similar pole, when
placed in vacuum (or air) at a distance of one metre from it,
with a force of 10–7 N.
6. The magnetic poles always exist in pairs. The poles of a
magnet can never be separated i.e. magnetic monopoles do
not exist.

20. MAGNETIC FIELD LINES


Magnetic field line is an imaginary curve, the tangent to
which at any point gives us the direction of magnetic field

B at that point.
If we imagine a number of small compass needless around a
magnet, each compass needle experiences a torque due to

Mahesh Tutorials Science


MAGNETISM 305

Properteis of magnetic field lines


1. The magnetic field lines of a magnet (or of a solenoid
carrying current) form closed continuous loops.
2. Outside the body of the magnet, the direction of magnetic
field lines is from north pole to south pole.
We shall show that the SI unit of M is joule/tesla or ampere
3. At any given point, tangent to the magnetic field line
 metre2.
represents the direction of net magnetic field ( B ) at that  SI unit of pole strength is Am.
point.
Bar magnet as an equivalent solenoid
4. The magnitude of magnetic field at any point is represented
We know that a current loop acts as a magnetic dipole.
by the number of magnetic field lines passing normally
According to Ampere’s hypothesis, all magnetic phenomena
through unit area around that point. Therefore, crowded
can be explained in terms of circulating currents.
lines represent a strong magnetic field and lines which are
not so crowded represent a weak magnetic field. In figure magnetic field lines for a bar magnet and a current
carrying solenoid resemble very closely. Therefore, a bar
5. No two magnetic field lines can intersect each other.
magnet can be thought of as a large number of circulating
currents in analogy with a solenoid. Cutting a bar magnet is
like cutting a solenoid. We get two smaller solenoids with
weaker magnetic properties. The magnetic field lines remain
continuous, emerging from one face of one solenoid and
entering into other face of other solenoid. If we were to
move a small compass needle in the neighbourhood of a bar
magnet and a current carrying solenoid, we would find that
21. MAGNETIC DIPOLE the deflections of the needle are similar in both cases.
To demonstrate the similarity of a current carrying solenoid
A magnetic dipole consists of two unlike poles of equal
to a bar magnet, let us calculate axial field of a finite solenoid
strength and separated by a small distance.
carrying current.
For example, a bar magnet, a compass needle etc. are
magnetic dipoles. We shall show that a current loop behaves
as a magnetic dipole. An atom of a magnetic material behaves
as a dipole due to electrons revolving around the nucleus.
The two poles of a magnetic dipole (or a magnet), called
north pole and south pole are always of equal strength, and
of opposite nature. Further such two magnetic poles exist
always in pairs and cannot be separated from each other.
T h e d i s t a n c e b e t w e e n t h e t w o p o l e s o f a b a r m a g n e t i s c a l l e d

magnetic length of the magnet. It is a vector directed from In figure, suppose



t h e

S-pole of magnet to its N-pole, and is represented by 2  . a = radius of solenoid,


Magnetic dipole moment is the product of strength of either 2 = length of solenoid with centre O

pole (m) and the magnetic length ( 2  ) of the magnet. n = number of turns per unit length of solenoid,
 i = strength of current passed through the solenoid
It is represented by M .
Magnetic dipole moment = strength of either pole × magnetic We have to calculate magnetic field at any point P on the
length axis of solenoid, where OP = r. Consider a small element of
thickness dx of the solenoid, at a distance x from O.
 
M  m 2   Number of turns in the element = n dx.
Using equation, magnitude of magnetic field at P due to this
Magnetic dipole moment is a vector quantity directed from current element is
South to North pole of the magnet, as shown in figure
0ia 2  n dx 
dB  3/ 2
2  r  x   a 2  ...(10)
2
 

Lakshya Educare
306 MAGNETISM

If P lies at a very large distance from O, i.e., r >> a and r >> x,


then [(r – x)2 + a2]3/2  r3
U  W   MB  cos 2  cos 1  ...(17)

1 = 90°, and 2 = , then


0ia 2 ndx
W h e n

dB  ...(11) U = W = – MB (cos  – cos 90°)


2r 3
W = – MB cos  ...(18)
As range of variation of x is from x = –  to x = +, therefore
the magnitude of total magnetic field at P due to current In vector notation, we may rewrie (18) as
carrying solenoid  
U   M.B ...(19)
2 x  
0 nia 0 nia 2
B  dx   x xx  
 
Particular Cases
2r 3 x  
2r 3 1. When  = 90°
U = – MB cos  = – MB cos 90° = 0
 ni a 2  2n  2  ia
2

3 
B 0 2   0 ...(12) i.e., when the dipole is perpendicular to magnetic field its potential
2 r 4 r3 energy is zero.
If M is magnetic moment of the solenoid, then Hence to calculate potential energy of diole at any position
M = total no. of turns × current × area of cross section making angle  with B, we use
M = n (2) × i × (a2) U = – MB (cos 2 – cos 1) and take 1 = 90° and 2 = .
Therefore,
0 2M U = – MB (cos  – cos 90°) = – MB cos 
 B ...(13)
4 r 3 2. When = 0°
This is the expression for magnetic field on the axial line of U = – MB cos 0° = – MB
a short bar magnet. which is minimum. This is the position of stable equilibrium,
Thus, the axial field of a finite solenoid carrying current is i.e., when the magnetic dipole is aligned along the magnetic
same as that of a bar magnet. Hence, for all practical purposes, field, it is in stable equilibrium having minimum P.E.
a finite solenoid carrying current is equivalent to a bar magnet. 3. When = 180°
Potential energy of a magnetic dipole in a magnetic field U = – MB cos 180° = MB, which is maximum. This is the
Potential energy of a magnetic dipole in a magnetic field is position of unstable equilibrium.
the energy possessed by the dipole due to its particular
position in the field.

When a magnetic dipole of moment M is held at an angle 

with the direction of a uniform magnetic field B , the
magnitude of the torque acting on the dipole is
  MBsin  ...(16)
This torque tends to align the dipole in the direction of the
field. Work has to be done in rotating the dipole against the
action of the torque. This work done is stored in the
magnetic dipole as potential energy of the dipole.
Now, small amount of work done in rotating the dipole
through a small angle d against the restoring torque is 22. MAGNETISM AND GAUSS’S LAW
dW = d= MB sin d
According to Gauss’s law for magnetism, the net magnetic
Total work done in rotating the dipole from = 1 to  = 2 is
flux (B) through any closed surface is always zero.
2
23. EARTH’S MAGNETISM
 MBsin  d  MB cos 12   MBcos 2  cos 1 

W
1
Magnetic elements of earth at a place are the quantities
 Potential energy of the dipole is which describe completely in magnitude as well as direction,
the magnetic field of earth at that place.

Mahesh Tutorials Science


MAGNETISM 307

Square (23) and (24), and add


23.1 Magnetic declination
H2 + V2 = R2 (cos2  + sin2 ) = R2
Magnetic declination at a place is the angle between
magnetic meridian and geographic meridian at that place.  R  H2  V2 ...(25)

Dividing (24) by (23), we get

R sin  V V
 or tan   ...(26)
R cos  H H
The value of horizontal component H = R cos  is different
at different places. At the magnetic poles,  = 90°
 H = R cos 90° = zero
At the magnetic equator,  = 0°
 H = R cos 0° = R
Horizontal component (H) can be measured using both, a
vibration magnetometer and a deflection magnetometer.
The value of H at a place on the surface of earth is of the
order of 3.2 × 10–5 tesla.
Retain in Memory
Memory note
1. The earth’s magnetic poles are not at directly opposite positions
on globe. Current magnetic south is farther from geographic Note that the direction of horizontal component H of earth’s
south than magnetic north is from geographic north. magnetic field is from geographic south to geographic north
above the surface of earth. (if we ignore declination).
2. Infact, the magnetic field of earth varies with position and
also with time. For example, in a span of 240 years from 1580 24. MAGNETIC PROPERTIES OF MATTER
to 1820 A.D., the magnetic declination at London has been
found to change by 3.5° – suggesting that magnetic poles To describe the magnetic properties of materials, we define
of earth change their position with time. the following few terms, which should be clearly understood
3. The magnetic declination in India is rather small. At Delhi,
declination is only 0° 41’ East and at Mumbai, the declination 24.1 Magnetic Permeability
is 0° 58’ West. Thus at both these places, the direction of
geographic north is given quite accurately by the compass It is the ability of a material to permit the passage of magnetic
needle (within 1° of the actual direction). lines of force through it i.e. the degree or extent to which magnetic
field can penetrate or permeate a material is called relative
23.2 Magnetic Dip or Magnetic Inclination magnetic permeability of the material. It is represented by r.
Relative magnetic permeability of a mterial is defined as the
Magnetic dip or magnetic inclination at a place is defined as ratio of the number of magnetic field lines per unit area (i.e.
the angle which the direction of total strength of earth’s flux density B) in that material to the number of magnetic
magnetic field makes with a horizontal line in magnetic meridian. field lines per unit area that would be present, if the medium
were replaced by vacuum. (i.e. flux density B0).
23.3 Horizontal Component
B
It is the component of total intensity of earth’s magnetic i.e., r 
field in the horizontal direction in magnetic meridian. It is B0
represented by H.
Relative magnetic permeability of a material may also be
In figure, AK represents the total intensity of earth’s magnetic defined as the ratio of magnetic permeability of the material
field, BAK = . The resultant intensity R along AK is () and magnetic permeability of free space (0)
resolved into two rectangular components :
Horizontal component along AB is 
 r  or    r  0
AL = H = R cos  ...(23) 0
Vertical component along AD is We know that 0 = 4 × 10–7 weber/amp-metre (Wb A–1 m–1)
AM = V = R sin  ...(24) or henry/metre (Hm–1)

Lakshya Educare
308 MAGNETISM

 SI units of permeability () are But B = H


Hm–1 = Wb A–1 m–1 = (T m2) A–1 m–1 = T m A–1 
 H  0 H 1   m  or  1  m
0

24.2 Magnetic Intensity ( H )
or r  1  m
The degree to which a magnetic field can magnetise a material
This is the relation between relative magnetic permeability
is represented in terms of magnetising force or magnetise
 and magnetic susceptibility of the material.
intensity ( H ).
25. CLASSIFICATION OF MAGNETIC MATERIALS
24.3 Magnetisation or Intensity of Magnetisation ‘I’
There is a large variety of elements and compounds on earth.
It represents the extent to which a specimen is magnetised, Some new elements, alloys and compounds have been
when placed in a magnetising field. Quantitatively, synthesized in the laboratory. Faraday classified these
The magnetisation of a magnetic material is defined as the substances on the basis of their magnetic properties, into
magnetic moment per unit volume of the material. the following three categories :
(i) Diamagnetic substances,
Magnetic moment m
M   (ii) Paramagnetic substances, and
volume V
(iii) Ferromagnetic substances
There are SI unit of I, which are the same as SI units of H.
Their main characteristics are discussed below :
Magnetic susceptibility (  m ) of a magnetic material is
25.1 Diamagnetic Substances
defined as the ratio of the intensity of magnetisation (I)
induced in the material to the magnetising force (H) applied The diamagnetic substances are those in which the
individual atoms/molecules/ions do not possess any net
on it. Magnetic susceptibility is represented by  m .
magnetic moment on their own. When such substances are
placed in an external magnetising field, they get feebly
I magnetised in a direction opposite to the magnetising field.
Thus  m 
H when placed in a non-uniform magnetic field, these
substances have a tendency to move from stronger parts of
Relation between magnetic permeability and magnetic
the field to the weaker parts.
susceptibility
When a specimen of a diamagnetic material is placed in a
When a magnetic material is placed in a magnetising field of
magnetising field, the magnetic field lines prefer not to pass
magnetising intensity H, the material gets magnetised. The through the specimen.
total magnetic induction B in the material is the sum of the
Relative magnetic permeability of diamagnetic substances
magnetic induction B0 in vacuum produced by the magnetic
is always less than unity.
intensity and magnetic induction Bm, due to magnetisation
of the material. Therefore, From the relation  r  1   m  , as  r  1,  m is negative.
B = B0 + Bm Hence susceptibility of diamagnetic substances has a small
But B0 = 0 H and Bm = m0 I, where I is the intensity of negative value.
magnetisation induced in the magnetic material. Therefore, A superconductor repels a magnet and in turn, is repelled
from above by the magnet.
The phenomenon of perfect diamagnetism in
B  0 H  0 I  0  H  I  , superconductors is called Meissner effect. Superconducting
magnets have been used for running magnetically leviated
i.e., B  0  H  I  superfast trains.

25.2 Paramagnetic substances


I
Now as  m   I  m H
H Paramagnetic substacnes are those in which each individual
atom/molecule/ion has a net non zero magnetic moment of
From above, B  0  H   m H   0 H 1   m  its own. When such substances are placed in an external

Mahesh Tutorials Science


MAGNETISM 309

magnetic field, they get feebly magnetised in the direction inversely proportional to the temperature (T) of the material.
of the magnetising field.
1
When placed in a non-uniform magnetic field, they tend to i.e., I  B, and I 
T
move from weaker parts of the field to the stronger parts.
When a specimen of a paramagnetic substance is placed in B
a magnetising field, the magnetic field lines prefer to pass Combining these factors, we get I 
T
through the specimen rather than through air.
As B  H , magnetising intensity
From the SI relation,  r  1   m , as  r  1 , therefore,  m
H I 1
must be positive. Hence, susceptibility of paramagnetic  I or 
substances is positive, though small. T H T
Susceptibility of paramagnetic substances varies inversely I
But  m
1 H
as the temperature of the substance i.e.  m  i.e. they
T
lose their magnetic character with rise in temperature. 1 C
 m  or m 
T T
25.3 Ferromagnetic substances where C is a constant of proportionality and is called Curie
Ferromagnetic substances are those in which each individual constant.
atom/molecule/ion has a non zero magnetic moment, as in a
paramagnetic substance. 26. HYSTERISIS CURVE
When such substances are placed in an external magnetising
The hysterisis curve represents the relation between
field, they get strongly magnetised in the direction of the field.  
magnetic induction B (or intensity of magnetization I ) of
The ferromagnetic materials show all the properties of
paramagnetic substances, but to a much greater degree. For a ferromagnetic material with magnetiziing force or magnetic
example, 
intensity H . The shape of the hysterisis curve is shown in
(i) They are strongly magnetised in the direction of external figure. It represents the behaviour of the material as it is
field in which they are placed. taken through a cycle of magnetization.
(ii) Relative magnetic permeability of ferromagnetic materials is 
Suppose the material is unmagnetised initially i.e., B  0
very large (  103 to 105)

(iii) The susceptibility of ferromagnetic materials is also very and H  0 . This state is represented by the origin O. Wee
large.  m  r  1 place the material in a solenoid and increase the current

That is why they can be magnetised easily and strongly. through the solenoid gradually. The magnetising force H
(iv) With rise in temperature, susceptibility of ferromagnetics 
increases. The magnetic induction B in the material
decreases. At a certain temperature, ferromagnetics change
increases and saturates as depicted in the curve oa. This
over to paramagnetics. This transition temperature is called
curie temperature. For example, curie temperature of iron is behaviour represents alignment and merger of the domains

about 1000 K. of ferromagnetic material until no further enhancement in B
is possible. Therefore, there is no use of inreasing solenoid
current and hence magnetic intensity beyond this.

25.4 Curie Law in Magnetism


According to Curie law,
Intensity of magnetisation (I) of a magnetic material is (i)
directly proportional to magnetic induction (B), and (ii)

Lakshya Educare
310 MAGNETISM

This phenomenon of lagging of I or B behind H when a


specimen of a magnetic material is subjected to a cycle of
magnetisation is called hysteresis.
For example, hysteresis loop for soft iron is narrow and
large, whereas the hysteresis loop for steel is wide and short,
figure

Next, we decrease the solenoid current and hence magnetic The hysterisis loops of soft iron and steel reveal that
 (i) The retentivity of soft iron is greater than the retentivity of
intensity H till it reduces to zero. The curve follows the
  steel,
path ab showing that when H  0 , B  0 . Thus, some
(ii) Soft iron is more strongly magnetised than steel,
magnetism is left in the specimen.
 (iii) Coercivity of soft iron is less than coercivity of steel. It
The value of magnetic induction B left in the specimen means soft iron loses its magnetism more rapidly than steel
when the magnetising force is reduced to zero is called does.
Retentivity or Remanence or Residual magnetism of the (iv) As area of I-H loop for soft iron is smaller than the area of
material. I-H loop for steel, therefore, hysterisis loss in case of soft
It shows that the domains are not completely randomised iron is smaller than the hysterisis loss in case of steel.
even when the magnetising force is removed. Next, the (a) Permanent Magnets
current in the solenoid is reversed and increased slowly.
Permanent magnets are the materials which retain at room
Certain domains are flipped until the net magnetic induction
 temperature, their ferromagnetic properties for a long time.
B inside is reduced to zero. This is represented by the The material chosen should have
curve bc. It means to reduce the residual magnetism or (i) high retentivity so that the magnet is strong,
retentivity to zero, we have to apply a magnetising force =
OC in opposite direction. This value of magnetising force is (ii) high coercivity so that the magnetisation is not erased by
called coercivity of the material. stray magnetic fields, temperature changes or mechanical
damage due to rough handling etc.
As the reverse current in solenoid is increased in magnitude,
we once again obtain saturation in the reverse direction at (iii) high permeability so that it can be magnetised easily.
d. The variation is represented by the curve cd. Next, the Steel is preferred for making permanent magnets.
solenoid current is reduced (curve de), reversed and (b) Electromagnets
increased (curve ea). The cycle repeats itself. From figure,
The core of electromagnets are made of ferromagnetic
we find that saturated magnetic induction BS is of the order
materials, which have high permeability and low retentivity.
of 1.5 T and coercivity is of the order of –90 Am–1.
Soft iron is a suitable material for this purpose. When a soft
From the above discussion, it is clear that when a specimen iron rod is placed in a solenoid and current is passed through
of a magnetic material is taken through a cycle of the solenoid, magnetism of the solenoid is increased by a
magnetisation, the intensity of magnetisation (I) and thousand fold. When the solenoid current is switched off,
magnetic induction (B) lag behind the magnetising force the magnetism is removed instantly as retentivity of soft
(H). Thus, even if the magnetising force H is made zero, the iron is very low. Electromagnets are used in electric bells,
values of I and B do not reduce to zero i.e., the specimen loudspeakers and telephone diaphragms. Giant
tends to retain the magnetic properties. electromagnets are used in cranes to lift machinery etc.

Mahesh Tutorials Science


MAGNETISM 311

Lakshya Educare
312 MAGNETISM

Specific example
27. HALL EFFECT
In the above circular loop tension in part A and B.
The Phenomenon of producing a transverse emf in a current
In balanced condition of small part AB of the loop is shown below
carrying conductor on applying a magnetic field perpendicular
to the direction of the current is called Hall effect.
Hall effect helps us to know the nature and number of charge
carriers in a conductor.
Consider a conductor having electrons as current carriers.
The electrons move with drift velocity v opposite to the
direction of flow of current

d d
2Tsin  dF  Bid  2Tsin  BiRd
2 2

d d d
If d is small so, sin   2T.  BiRd
 
 
2 2 2
Force acting on electron Fm   e v  B . This force acts
BiL
along x-axis and hence electrons will move towards face (2) T  BiR, if 2R  L so T 
and it becomes negatively charged. 2

28. STANDARD CASES FOR FORCE ON


CURRENT CARRYING CONDUCTORS
Case 1 : When an arbitrary current carrying loop placed in
If no magnetic field is present, the loop will still open into
a magnetic field (  to the plane of loop), each element of
a circle as in it’s adjacent parts current will be in opposite
loop experiences a magnetic force due to which loop
direction and opposite currents repel each other.
stretches and open into circular loop and tension developed
in it’s each part.

Case 2 : Equilibrium of a current carrying conductor :


When a finite length current carrying wire is kept parallel to
another infinite length current carrying wire, it can suspend
freely in air as shown below

Mahesh Tutorials Science


MAGNETISM 313

Wire is placed along the axis of coil so magnetic field


produced by the coil is parallel to the wire. Hence it will not
experience any force.
Case 4 : Current carrying spring : If current is passed
through a spring, then it will contract because current will
flow through all the turns in the same direction.

In both the situations for equilibrium of XY it’s downward


0 2i1i 2
weight = upward magnetic force i.e. mg  . .
4 h

* In the first case if wire XY is slightly displaced from its


equilibrium position, it executes SHM and it’s time period
If current makes to flow through spring, then spring will
h contract and weight lift up.
is given by T  2 .
g

* If direction of current in movable wire is reversed then


it’s instantaneous acceleration produced is 2g.

Case 3 : Current carrying wire and circular loop : If a


current carrying straight wire is placed in the magnetic field
of current carrying circular loop.

If switch is closed then current start flowing, spring will


execute oscillation in vertical plane.
Case 5 : Tension less strings : In the following figure the
value and direction of current through the conductor XY so
that strings becomes tensionless ?
Strings becomes tensionless if weight of conductor XY
balanced by magnetic force (Fm).
Wire is placed in the perpendicular magnetic field due to
coil at it’s centre, so it will experience a maximum force
0i1
F  Bi   i 2
2r

Lakshya Educare
314 MAGNETISM

In the following situation conducting rod (X, Y) slides at


constant velocity if

mg
Fcos   mg sin   Bi cos   mg sin   B  tan 
i

TIPS & TRICKS


1. The device whose working principle based on Halmholtz
Hence direction of current is from X  Y and in balanced coils and in which uniform magnetic field is used called as
mg “Halmholtz galvanometer”.
condition Fm = mg  Bi = mg  i =
B 2. The value of magnetic field induction at a point, on the
centre of separation of two linear parallel conductors
Case 6 : A current carrying conductor floating in air such
carrying equal currents in the same direction is zero.
that it is making an angle  with the direction of magnetic
field, while magnetic field and conductor both lies in a 3. If a current carrying circular loop (n = 1) is turned into a
horizontal plane. coil having n identical turns then magnetic field at the
centre of the coil becomes n2 times the previous field i.e.
B

(n turn)
= n2 B(single turn).
4. When a current carrying coil is suspended freely in earth’s
magnetic field, it’s plane stays in East-West direction.

5.  
Magnetic field B produced by a moving charge q is given

 0 q  v  r  0 q  v  rˆ 
mg by B   ; where v = velocity of
In equilibrium mg = Bi sin  i  4 r 3 4 r 2
B sin 
charge and v < < c (speed of light).
Case 7 : Sliding of conducting rod on inclined rails : When
a conducting rod slides on conducting rails.

6. If an electron is revolving in a circular path of radius r with


speed v then magnetic field produced at the centre of circular

0 ev v
path B  . r .
4 r 2 B

7.  
The line integral of magnetising field H for any closed
path called magnetomotive force (MMF). It’s S.I. unit is amp.
8. Ratio of dimension of e.m.f. to MMF is equal to the dimension
of resistance.
9. The positive ions are produced in the gap between the two
dees by the ionisation of the gas. To produce proton,
hydrogen gas is used; while for producing alpha-particles,
helium gas is used.

Mahesh Tutorials Science


MAGNETISM 315

10. Cyclotron frequency is also known as magnetic resonance


frequency.
11. Cyclotron can not accelerate electrons because they have
very small mass.
12. The energy of a charged particle moving in a uniform magnetic
field does not change because it experiences a force in a
direction, perpendicular to it’s direction of motion. Due to
which the speed of charged particle remains unchanged and
hence it’s K.E. remains same.
17. If no magnetic field is present, the loop will still open into a
13. Magnetic force does no work when the charged particle is circle as in it’s adjacent parts current will be in opposite
displaced while electric force does work in displacing the
direction and opposite currents repel each other.
charged particle.
14. Magnetic force is velocity dependent, while electric force
is independent of the state of rest or motion of the charged
particle.
15. If a particle enters a magnetic field normally to the
magnetic field, then it starts moving in a circular orbit.
The point at which it enters the magnetic field lies on the
circumference. (Most of us confuse it with the centre of the
orbit)
16. Deviation of charged particle in magnetic field : If a
 18. In the following case if wire XY is slightly displaced from its
charged particle (q, m) enters a uniform magnetic field B equilibrium position, it executes SHM and it’s time period is
(extends upto a length x) at right angles with speed v as
shown in figure. The speed of the particle in magnetic h
given by T  2 .
field does not change. But it gets deviated in the magnetic g
field.

 Bq 
Deviation in terms of time t ;   t   t
 m

Deviation in terms of length of the magnetic field ;

x
  sin 1   . This relation can be used only when x  r .
r

For x > r, the deviation will be 180° as shown in the following figure 19. In the previous case if direction of currnet in movable wire
is reversed then it’s instantaneous acceleration produced is
2g.
20. Electric force is an absolute concept while magnetic force is
a relative concept for an observer.
21. The nature of force between two parallel charge beams
decided by electric force, as it is dominator. The nature of
force between two parallel current carrying wires decided
by magnetic force.

Lakshya Educare
316 MAGNETISM

24. If a current carrying conductor AB is placed transverse to a


22. If a straight current carrying wire is placed along the axis of long current carrying conductor as shown then force.
a current carrying coil then it will not experience magnetic
Experienced by wire AB
force because magnetic field produced by the coil is parallel
to the wire. 0i1i 2  x
F log e  
23. The force acting on a curved wire joining points a and b as 2  x 
shown in the figure is the same as that on a straight wire
  
joining these points. It is given by the expression F  iL  B

Mahesh Tutorials Science


MAGNETISM 317

SOLVED EXAMPLES

Example - 1 mv m
r or r 
An electron is passing through a field but no force is acting Bq q
on it. Under what conditions is it possible, if the motion of
the electron be in the (i) electric field (ii) magnetic field ? rp mp q   m   2e  1
     
r m q p  4m   e  2
Sol. (i) In electric field, there is always a force on the moving
electron opposite to the direction of field. Thus the force
or r  2rp i.e. r  rp .
will be zero only if electric field is zero.
(ii) In magnetic field, the force acting on a moving electron is i.e. track B corresponds to -particle and track A to proton.
F = qv B sin , it is zero if  = 0º or 180º. Example - 3
i.e. the electron is moving parallel to the direction of magnetic Why is ammeter connected in series and voltmeter in
field. parallel in the circuit ?

Example - 2 Sol. An ammeter is a low resistance galvanometer. It is used to


A neutron, a proton an electron and an -particle enter a measure the current in ampere. To measure the current of a
region of constant magnetic field with equal velocities. circuit, the ammeter is connected in series to the circuit so
that the current to be measured must pass through it. Since,
The magnetic field is along the inward normal to the plane
the resistance of ammeter is low, so its inclusion in series in
of paper. The tracks of the particles are shown in figure.
the circuit does not change the resistance and hence the
Relate the tracks to the particles.
main current in the circuit.
A voltmeter is a high resistance galvanometer. It is used to
measure potential difference between two points of the
circuit in volt. To measure the potential difference between
the two points of a circuit, the voltmeter is connected in
parallel to the circuit. The voltmeter resistance being high, it
draws minimum current from the main circuit and the potential
difference to be measured is not affected materially.
Example - 4
Sol. We know that force on a charged particle in the magnetic A current carrying circular loop is located in a uniform
field is external magnetic field. If the loop is free to turn, what is its
   orientation of stable equilibrium? Show that in this
F  q v  B or F  qvBsin , so orientation, the flux of the total field (external field + field
(i) For neutral particle i.e. neutron, q = 0, hence F = 0. It means produced by the loop) is maximum.
neutron will go undeflected i.e. track C corresponds to Sol. The current carrying circular loop behaves as a magnetic
neutron. 
dipole of magnetic moment M acting perpendicular to its
(ii) For negatively charged particle i.e. electron, the direction of plane. The torque on the current loop of magnetic dipole
force, according to Fleming’s Left hand rule will be towards moment M in the magnetic field B is
right. So track D corresponds to electron.
 = MB sin  = IA × B sin , ( M = AI)
(iii) For positively charged particle, the direction of force,  
where  is the angle between M and B . The system will be
according to Fleming’s left hand rule will be towards left. So
in stable equilibrium if torque is zero, which is so if  = 0º.
both tracks A and B correspond to positively charged   
particles (i.e. protons and -particles). This is possible if B is parallel to A i.e. B is perpendicular
When a moving charged particle is subjected to a to the plane of the loop. In this orientation, the magnetic
perpendicular magnetic field, it describes a circular path of field produced by the loop is in the same direction as that of
radius r given by external field, both normal to the plane of loop. It is due to
this fact, the magnetic flux due to total field is maximum.

Lakshya Educare
318 MAGNETISM

Example - 5 electric field time and again with the use of strong magnetic
Three wires each of length 2.0 m are bent into different field.
rectangular loops and then suspended in a magnetic field, A cyclotron is used (i) to bombard nuclei with high energy
figure. If the current in each of them be the same, which particles and to study the resulting nuclear reaction (ii) to
loop shall be acted upon by largest torque ? If any of the produce radioactive substances which may be used in
wires be bent into circular loop, then ? hospitals for diagnosing the diseases in the body.
Example - 9
A charged particle enters into a uniform magnetic field and
experiences upwardforce as indicated in figure. What is
the charge sign on the particle ?

Sol. Torque () on a current loop suspended in a uniform magnetic Sol. The particle has a positive charge.
field is given by = I AB sin  i.e.   A. Since the area of
Example - 10
loop (c) = 0.5 m × 0.5 m is maximum; hence the largest torque
will be acting on it. When any wire is bent into a circular You are given a low resistance R1, a high resistance R2
loop, the torque will be even more because for a given and a moving coil galvanometer. Suggest how you would
perimeter the area of the circle is maximum. use these to have an instrument that will be able to
measure (i) currents (ii) potential differences.
Example - 6
What is meant by cyclotron frequency ? Sol. (i) To measure currents, the low resistance R1 is connected
in parallel to the moving coil galvanometer.
Sol. It is the frequency of oscillation of a heavy charged particle
(ii) To measure potential differences, a high resistance R2 is
in between two dees of cyclotron, which is equal to the
connected in series with the moving coil galvanometer.
frequency of high frequency oscillator, creating electric field
between two dees of cyclotron. Cyclotron frequency, Example - 11
v = Bq/2  m, which is independent of the radius of the State properties of the material of the wire used for
circular path and velocity of the charged particle in the two suspension of the coil in a moving coil galvanometer.
dees of cyclotron. Sol. The properties of the material of the wire used for suspension
Example - 7 of the coil in a moving coil galvanometer are as follows :

 
A charge 3 coulomb is moving with velocity v  4iˆ  3jˆ ms
1 1. It should have low torsional constant i.e. restoring torque
per unit twist should be small.

 2
in a magnetic field B  4iˆ  3jˆ Wbm . Find the force 2. It should have high tensile strength.
acting on the charge. 3. It should be a non-magnetic substance.
  
   
4. It should have a low temperature coefficient of resistance.
 
Sol. F  q v  B  3  4iˆ  3jˆ  4iˆ  3jˆ  = 3 [0] = 0
5. It should be a good conductor of electricity.
 Cross product of two equal vector is zero. Example - 12
Example - 8 What is a radial magnetic field ? How has it been achieved
What is the basic principle of working of cyclotron ? Write in moving coil galvanometer ?
two uses of this machine. Sol. Radial magnetic field is that field, in which the plane of the
Sol. The working of the cyclotron is based on the fact that a coil always lies in the direction of the magnetic field. A radial
heavy positively charged particle can be accelerated to a magnetic field has been achieved by (i) properly cutting the
sufficiently high energy with the help of smaller values of magnetic pole pieces in the shape of concave faces. (ii)
oscillation electric field, by making it to cross the same using a soft iron core within the coil.

Mahesh Tutorials Science


MAGNETISM 319

Example - 13 Example - 16
Why is phosphor bronze alloy preferred for the suspension An electron and proton enter perpendicularly in a uniform
wire of a moving coil galvanometer ? magnetic field with the same speed. How many times larger
will be the radius of proton’s path than the electron’s ?
Sol. The suspension wire of phosphor bronze alloy is preferred in Proton is 1840 times heavier than electron.
moving coil galvanometer because it has several advantages:
(i) Its restoring torque per unit twist is small. Due to it, the Sol. The charged particle while moving perpendicular to magnetic
galvanometer is very sensitive. field experiences a force which provides the centripetal force
for its circular motion. The radius r of the circular path traced
(ii) It has great tensile strength so that even if it is thin, it will not
by the particle in magnetic field B, is given by Bqv = mv2/r or
break under the weight of the coil suspended from its end.
r = mv/Bq or r  m if v, B and q are constant.
(iii) It is rust resisting. Hence it remains unaffected by the weather
Since the value of charge on electron and proton is the
conditions of air in which it is suspended.
same but mass of proton is 1840 times mass of electron,
Example - 14
rp mp 1840m e
What is the main function of a soft iron core used in a hence    1840 or r = 1840 r .
re me me p e
moving coil galvanometer ?
Example - 17
Sol. (i) This makes the magnetic field radial. In such a magnetic
field the plane of the coil is always parallel to the direction Two parallel wires carrying current in the same direction
of magnetic field. Due to which the galvanometer scale attract each other while two beams of electrons travelling
becomes linear. in the same direction repel each other. Why ?

(ii) This increases the strength of magnetic field due to the


Sol. Two parallel wires carrying currents in the same direction
crowding of the magnetic lines of force through the soft
attract each other due to magnetic interaction between two
iron core, which in turn increases the sensitiveness of the
wires carrying currents because the current in a wire
galvanometer.
produces a magnetic field and the magnetic interaction is of
Example - 15 attractive nature when current is the two parallel wires is in
Define current sensitivity and voltage sensitivity of a the same direction.
galvanometer. Increase in the current sensitivity may not The two beams of electrons travelling in the same direction
necessarily increase the voltage sensitivity of a will be a source of both an electric and magnetic fields. Due
galvanometer. Justify. to magnetic interaction, there will be force of attraction
between the two moving electrons but due to electrostatic
Sol. For definition of current sensitivity and voltage sensitivity
interaction, there will be a force of repulsion between them.
refer to Art. 3(b).11.
If the beams of electrons are moving slowly, the electrostatic
Let  be the deflection produced in the galvanometer on force of repulsion between the electrons dominates the
applying voltage V, then magnetic attraction between them.
 nBA Example - 18
current sensitivity  
I k An electron beam moving with uniform velocity is
 nBA gradually diverging. When it is accelerated to a very high
voltage sensitivity   velocity, it again starts converging. Why ?
V kR
Thus, the current sensitivity can be increased by increasing,
Sol. Moving electrons, apart from electrical repulsion experience
n, B, A and by decreasing k. If n is increased, it will increase
magnetic attraction also. If the electron beam is moving
the resistance of conductor. under normal conditions, the electrical repulsive force is
The voltage sensitivity can be increased by increasing n, B, much stronger than the magnetic attraction and hence the
A and by decreasing k and R. beam diverges. When the electron beam is moving at very
Therefore, the increase in current sensitivity of galvanometer high velocity, the magnetic force of attraction becomes more
may not necessarily increase the voltage sensitivity of the effective than electrical repulsion and the beam starts
galvanometer. converging.

Lakshya Educare
320 MAGNETISM

Example - 19 Example - 21
Distinguish between Biot Savart’s law and Ampere’s Two parallel coaxial circular coils of equal radius R and
circuital law. equal number of turns N carry equal currents I in the same
Sol. direction and are separated by a distance 2 R. Find the
magnitude and direction of the net magnetic field
Biot-Savart’s Law Ampere’s Circuital Law
produced at the mid-point of the line joining their centres.
1. This law is based on the This law is based on the
principle of magnetism. principle of electromagnetism. Sol. Magnetic field induction at the mid-point due to current
2. This law is valid for This law is valid for loop 1 is
asymmetrical current symmetrical current 0 2I R 2 0 I R 2
distribution. distributions. B1   , acting towards right.
3. This law is the differential This law is the integral form  
4 R 2  R 2 3/ 2 2 2R 2 3/ 2
 
 
form of magnetic field of B or H . Magnetic field induction at the mid point due to current
 loop 2 is
induction B or
 0 I R 2 0 I R 2
magnetising force H . B2   , acting towards right.
   
3/ 2 3/ 2
2 R2  R2 2 2R 3
Example - 20
Total magnetic field induction is
Two small circular loops, marked (1) and (2), carrying equal
currents are placed with the geometrical axes perpendicular 0 I R 2 0 I R 2 0 I R 2 0 I
B  B1  B2    
   
to each other as shown in figure. Find the magnitude and 2 3/ 2 2 3/ 2 2 2R 3
2 2R
2 2R 2 2R
direction of the net magnetic field produced at the point O.
Example - 22
Magnetic field lines can be entirely confined within the
core of a toroid, but not within a straight solenoid. Why ?

Sol. It is so because the magnetic field idnuction outside the


toroid is zero.
Example - 23
Name the physical quantity whose unit is tesla. Hence
define a tesla.

Sol. Magnetic field induction at O due to current loop 1 is Sol. Tesla is the SI unit of magnetic field induction or magnetic
flux density at a point in the magnetic field. The magnetic
0 I R 2 field induction at a point in a magnetic field is said to be 1
B1  , acting towards left.
  tesla if one coulomb charge while moving with a velocity of
3/ 2
2 x2  R 2
1 m/s, perpendicular to the magnetic field experiences a force
Magnetic field induction at O due to current loop 2 is of 1 N at that point.
Example - 24
0 I R 2
B2  acting vertically upwards. What is meant by a magnetic field ? How is it produced ?
 
3/ 2
2 x2  R2
Sol. A magnetic field is the space around a magnet or the space
Resultant magnetic field induction at O will be around a wire carrying current, in which its magnetic effect
can be felt.
B  B12  B22  2 B1  B1  B2 
A magnetic field may be produced in many ways. For
example, (i) by a magnet (ii) by a current carrying conductor
0 I R 2 0 I R 2
 2  (iii) by a moving charge (iv) by a varying electric field.
 
2 3/ 2
 
3/ 2
2 x2  R 2 x2  R2 (displacement current)

Mahesh Tutorials Science


MAGNETISM 321

Example - 25 Example - 30
What is the potential energy of a dipole when it is State two methods to destroy the magnetism of a magnet.
perpendicular to a magnetic field ?
Sol. (i) By heating the magnet.
Sol. P.E. = –MB cos  = –MB cos 90º = zero. (ii) By applying magnetic field in the reverse direction.
Example - 26 Example - 31
What is the basic difference between magnetic and electric An electron of energy 2000 eV describes a circular path in
lines of force ? magnetic field of flux density 0.2 T. What is the radius of
the path ? Take e = 1.6 × 10–19 C, m = 9 × 10–31 kg.
Sol. Magnetic lines of force are closed, continuous curves, but
Sol. Here, energy of electron, E’ = 2000 eV
electric lines of force are discontinuous.
= 2000 × 1.6 × 10–19 J = 3.2 × 10–16 J.
Example - 27
B = 0.2 T ; r = ?
A magnetic needle free to rotate in a vertical plane, orients
itself with its axis vertical at a certain place on the earth. 1 2E '
What are the values of As, E'  mv 2  v
2 m
(a) Horizontal component of earth’s field ?
mv2
(b) angle of dip at this place. Also, Bev 
r
Sol. H = 0 and  = 90º. The place will be magnetic pole of earth.
mv m 2E ' 2E 'm
or r  
Example - 28 Be Be m Be
Why do magnetic lines of force prefer to pass through iron
2  3.2  1016  9  1031
than air ?   7.5  104 m
0.2  1.6  1019
Sol. This is because permeability of soft iron is much greater Example - 32
than that of air.
A long straight wire AB carries a current of 4 A. A proton P
travels at 4 × 106 ms–1 parallel to the wire, 0.2 m from it and
Example - 29 in a direction opposite to the current as shown in figure.
Define the term : magnetic dipole moment of a current loop. Calculate the force which the magnetic field of current
Write the expression for the magnetic moment when an exerts on the proton. Also specify the direction of the force.
electron revolves at a speed v around an orbit of radius r in
hydrogen atom.

Sol. A current carrying loop behaves as a system of two equal


and opposite magnetic poles separated by a distance. Hence
it behaves as a magnetic dipole. Magnetic dipole moment of
current loop is the product of current I and area A enclosed
by the loop of current, i.e. M = IA.
In a hydrogen atom, when an electron revolves at a speed v
Sol. Here, I = 4A ; v = 4 × 106 ms–1 ; a = 0.2 m.
around an orbit of radius r, the magnetic moment is given by
Magnetic field induction at P is
 eh 
M  n  0 2I 107  2  4
 4m  B   4  106 T
4 r 0.2
where e is charge on electron, m is mass of electron ; 
n denotes the number of orbit and h is Plack’s constant. The direction of B , according to Right Hand Thumb rule is
perpendicular to the plane of paper directed inwards.

Lakshya Educare
322 MAGNETISM

Since proton is moving in opposite direction to the current


E 2  104
carrying straight wire, hence the proton is moving eE  evB or v    5  106 m / s
perpendicular to the direction of magnetic field due to current B 4  103
through straight wire. The force on moving proton of charge When electron moves perpendicular to magnetic field, the
q due to magnetic field is radius r of circular path traced by electron is
F = qvB sin 90º = (1.6 × 10–19) × (4 × 106) × (4 × 10–6)
= 2.56 × 10–18 N
r
mv


9.1 1031  5  106   
 7.11 103 m  7.11 mm
The direction of force on proton, according to Fleming’s eB 
1.6  1019  4  103 
Left Hand Rule acts in the plane of paper towards right.

Example - 33 Example - 35
Figure shows a rectangular current-carrying loop placed
A cyclotron oscillator frequency is 10 M Hz. What should be
2 cm away from a long, straight, current carrying conductor.
the operating magnetic field for accelerating -particle ? If
What is the direction and magnitude of the net force acting
the radius of the dees is 50 cm, what is the kinetic energy in
on the loop ?
MeV of the -particle beam produced by the accelerator?
(e = 1.6 × 10–19 C ; m = 4.0028 a.m.u. ; 1 a.m.u. = 1.66 × 10–27 kg)

Sol. Here, v = 10 MHz = 107 Hz ; r0 = 50 cm = 0.50 m ; B = ?


m = 4.0028 × 1.66 × 10–27 kg = 6.645 × 10–27 kg,
q = 2 e = 2 × 1.6 × 10–19 = 3.2 × 10–19 C.

Bq
As, v
2m

2m v
or B
q

22 6.645  1027  107


 2   1.305 T
7 3.2  1019 Sol. Here, I1 = 15 A ; I2 = 25 A ;
Maximum kinetic energy is r1 = 2 × 10–2 m ; r2 = (2 + 10) × 10–2 m

 
2
B2 q 2 r 2 1.305  3.2  10   0.50 
19
2 2
0 2I1I 2
E max   J Force on BC, F1   length BC
4 r1
2m 
2  6.645  1027 
2  15  25
1.3052   3.2 2 1038  0.25  107  
 25  102 
 MeV  20.5 MeV  2  102 
2  6.645  1027  1.6  1013
= 9.375 × 10–4 N (repulsive, away from XY)
Example - 34
An electron beam passes through a magnetic field of 4 × 10–3 0 2I1I 2
weber/m2 and an electric field of 2 × 104 Vm–1, both acting Force on DA, F2   length DA
4 r2
simultaneously. The path of electron remaining undeviated,
calculate the speed of the electrons. If the electric field is 2  15  25
removed, what will be the radius of the electron path ?  107   25  102
 2  10  10 2

Sol. Here, B = 4 × 10–3 weber/m2 ; E = 2 × 104 V/m.


= 1.5625 × 10–4 N (attractive towards XY)
As the path of moving electron is undeviated, so force on
Net force on the loop F = F1 – F2 = (0.375 – 1.5625) × 10–4
moving electron due to electric field is equal and opposite
to the force on moving electron due to magnetic field i.e. = 7.8175 × 10–4 N (respulsive, away from XY)

Mahesh Tutorials Science


MAGNETISM 323

Example - 36   
(iii)   M  B  MBsin  where  is the angle between M
A long straight conductor PQ, carrying a current of 60 A, is 
 
fixed horizontally. Another long conductor XY is kept and B or between A and B .
parallel to PQ at a distance of 4 mm, in air. Conductor XY is Initially,  = 0º,  = MB sin 0º = 0.
free to move and carries a current I. Calculate the magnitude
Finally,  = 90º,  = MB sin 90º = MB = 10 × 2 = 20 Nm.
and direction of current I for which the magnetic repulsion
just balances the weight of conductor XY. (Mass per unit d d d d
(iv)   I  I  I   I    MBsin 
lengths for conductor XY is 10–2 kg/m). dt d dt d
 Id  MBsin d .
Integrating it within the given conditions,
 /2
I   d   MBsin  d
0 0

2 /2   
Sol. Here, I1 = 60 A ; I2 = I A, r = 4 mm = 4 × 10–3 m ; I  MB   cos  0   MB cos  cos 0º   MB
Mass per unit length of conductor XY, m = 10–2 kg/m. 2  2 
As magnetic repulsion is balancing the weight of conductor 1/ 2 1/ 2
 2MB   2  20 
XY or      20 rad / s.
 I   0.1 
0 2I1I 2 107  2  60  I Second Method for (iv)
so,  mg or  102  9.8
4 r 4 103 Change in KE of rotation = work done in rotation

4  105  9.8 1 2
or I  32.67 A I  MB  cos 1  cos 2  where
2  107  60 2
The current in XY must flow opposite to that in PQ, because
1  0º ; 2  90º , I  0.1 kg m2 ; MB  20 Nm
only then the force will be repulsive.

 2MB  cos 1  cos 2  


1/ 2
Example - 37
   
A 100 turn closely wound circular coil of radius 10 cm  I 
carries a current of 3.2 A. (i) What is the field at the centre of
 2  20   cos 0º  cos90º  
1/ 2
the coil ? (ii) What is the magnetic moment of this
arrangement ? The coil is placed in a vertical plane and is    20 rad / s
 0.1 
free to rotate about a horizontal axis which coincides with
its diameter. A uniform magnetic field of 2 T in the horizontal Example - 38
direction exists such that initially the axis of the coil is in
A circular coil of 100 turns, radius 10 cm carries a current of
the direction of the field. The coil rotates through an angle 5 A. It is suspended vertically in a uniform horizontal
of 90º under the influence of the magnetic field. (iii) What
magnetic field of 0.5 T, the field lines making an angle of
are the magnitudes of the torques on the coil in the initial
60º with the plane of coil. Calculate the magnitude of the
and final position ? (iv) What is the angular speed acquired torque that must be applied on it to prevent it from turning.
by the coil when it has rotated by 90º ? The moment of
inertia of the coil is 0.1 kg m2. Sol. Here, n = 100 ; I = 5 A ; B = 0.5 T ;  = 90º – 60º = 30º ; r = 10
cm = 0.10 m ;
Sol. (i) Here, n = 100, r = 0.10 m, i = 3.2 A, B = 2 T, I = 0.1 kg m2
22
  0.10  m 2
2
 2ni 22 100  3.2 A  r 2 
B 0  107  2    2  103 T 7
4 r 7 0.10
22
Torque,  = nIBA sin  = 100 × 5 × 0.5 × × (0.10)2 × sin 30º
 22  7
M = niA = nir2 = 100   3.2       0.10   10 Am2
2
(ii) = 3.927 N-m
 7 

Lakshya Educare
324 MAGNETISM

Example - 39 Vs  Vs'
Compare the current sensitivity and voltage sensitivity of  % decrease in voltage sensitivity   100
Vs
the following moving coil galvanometers :
Meter A : n = 30, A = 1.5 × 10–3 m2, B = 0.25 T, R = 20  3
Vs  Vs
 5  100  40%
Meter B : n = 35, A = 2.0 × 10–3 m2, B = 0.25 T, R = 30  Vs
You are given that the springs in the two metres have the
Example - 41
same torsional constants.
A galvanometer having 30 divisions has a current
Sol. For metre A, n1 = 30, A1 = 1.5 × 10–3 m2, B1 = 0.25 T, R1 = 20 . sensitivity of 20 A/division. It has a resistance of 25 .
For metre B, n2 = 35, A2 = 2.0 × 10–3 m2, B2 = 0.25 T, R2 = 30 . How will you convert it into an ammeter upto 1 ampere ?
How will you convert this ammeter into a voltmeter up to 1
nBA
Current sensitivity of a meter  volt ?
k

Current sensitivity of meter A Sol. Current sensitivity = 20 A/div. = 20 × 10–6 A/div.


 Current sensitivity of meter B Current for full scale deflection, ig = 20 × 10–6 × 30
= 6 × 10–4 A
nBA k2 nBA
 1 1 1  1 1 1
n 2 B2 A 2 n 2 B2 A 2  k1  k 2 
For converting galvanometer into ammeter the shunt required
k1
Ig 6  104  25
30  0.25  1.5  103 45 9 S .G   0.1050 
   I  Ig 1  6  104
35  0.25  2.0  103 70 14

nBA Resistance of ammeter formed,


Now, voltage sensitivity of a meter 
kR GS 0.015  25
G'    0.015 
Voltage sensitivity of meter A G  S 25  0.015
 Voltage sensitivity of meter B Conversion of ammeter into voltmeter
Here, Ig = 1 ampere, V = 1 volt, G’ = 0.015 
n1B1A1 k R nBAR
  2 2  1 1 1 2
k1R1 n 2 B2 A 2 n 2 B2 A 2 R1 Resistance to be used in series,

V 1
30  0.25  1.5  103  30 9 30 27 R  G '   0.015  0.985 
    Ig 1
35  0.25  2.0  103  20 14 20 28
Example - 42
Example - 40
A resistance of 1980  is connected in series with a
The current sensitivity of a moving coil galvanometer
increases by 20% when its resistance is increased by a factor voltmeter, after which the scale division becomes 100 times
2. Calculate by what factor the voltage sensitivity changes. larger. Find the resistance of voltmeter.

Sol. Let R be the resistance of voltmeter. Let n be the number of


20 120 divisions in the voltmeter. The voltage recorded by each
Sol. Given, Is'  Is  Is  Is ; R '  2R
100 100 division of voltmeter when current ig flows through it is
ig R/n = V ...(i)
Is
Then, initial voltage sensitivity, Vs  when resistance is connected in series of voltmeter then
R
ig (R + 1980)/n = 100 V ...(ii)
New voltage sensitivity,
Dividing (ii) by (i), we get
I'  120  1 3 R + 1980 = 100 R
Vs'  s   Is    Vs
R '  100  2R 5
or R = 1980/99 = 20 

Mahesh Tutorials Science


MAGNETISM 325

Example - 43 Example - 46
43. A magnetised steel wire 31.4 cm long has a pole strength of In the magnetic meridian of a certain place, the horizontal
0.2 Am. It is then bent in the form of a semicircle. Calculate component of the earth’s magnetic field is 0.26 G and dip
magnetic moment of the needle. angle is 60º. What is the magnetic field of earth at this
location ?
Sol. Here, L = 31.4 cm. m = 0.2 Am, M = ? Sol. Here, H = 0.26 G,  = 60º, R = ?
When the wire is bent in the form of a semicircle of radius r, As H = R cos 
then L = r = 3.14 r
H 0.26 0.26
 R    0.52 G
cos  cos 60º 1/ 2 
L 31.4
r   10 cm
3.14 3.14
Example - 47
Distance between the two ends of wire,
2 = 2r = 20 cm = 0.2 m A magnetic needle has magnetic moment of 6.7 × 10–2 Am2
and moment of inertia of 7.5 × 10–6 kg m2. It performs 10
M = m × 2 = 0.2 × 0.2 = 0.04 Am2 complete oscillations in 6.70 s. What is the magnitude of
Example - 44 the magnetic field ?
A magnetised needle of magnetic moment 4.8 × 10–2 J T–1 is Sol. Here, M = 6.7 × 10–2 Am2, I = 7.5 × 10–6 kg m2
placed at 30º with the direction of uniform magnetic field of
magnitude 3 × 10–2 T. What is the torque acting on the 6.70
needle ? Time for one oscillation, T   0.67 s ; B  ?
10

Sol. Here, M = 4.8 × 10–2 J T–1 ;  = 30º ; B = 3 × 10–2 T I 4 2 I


From T*  2 ;B
torque,  = ? MB MT 2
As  = MB sin 
4   22 / 7   7.5  10 6
2
  = 4.8 × 10–2 × 3 × 10–2 sin 30º   0.01 T
6.7  102  0.67 
2
= 7.2 × 10–4 N-m
Example - 45 Example - 48
A ship is to reach a place 10º south of west. In what The core of a toroid having 3000 turns has inner and outer
direction should it be steered if declination at the place is radii 11 cm and 12 cm respectively. Calculate relative
17º west ? permeability of its core, given that a current of 0.7 amp.
produces a magnetic field of intensity 2.5 T in the core.
Sol. As the ship is to reach a place 10º south of west i.e. along
OA, figure, therefore, it should be steered west of (magnetic) Sol. Here, total number of turns = 3000
north at an angle of (90 – 17 + 10) = 83º.
11  12
Average radius, r   11.5 cm
2
= 11.5 × 10–2 m ; i = 0.7 amp. and B = 2.5 T
No. of turns/length,

3000 3000 3  105


n  
2r 2 11.5  102 23

As B=ni
B = 0 r n i

B 2.5  23
 r    684.5
0 ni 4 107  3  105  0.7

Lakshya Educare
326 MAGNETISM

Example - 49 (a) 2iBL directed along the negative Z-axis


–5
The susceptibility of magnesium at 300 K is 1.2 × 10 . (b) 5iBL directed along the positive Z-axis
At what temperature will the susceptibility be equal to
(c) iBL direction along the positive Z-axis
1.44 × 10–5 ?
(d) 2iBL directed along the positive Z-axis

C m T ' Sol : (c)


Sol. As  m  T  '  T
m As PQ and UT are parallel to Q, therefore FPQ = FUT = 0

m 1.2  105 The current in TS and RQ are in mutually opposite direction.


or T'  T   300  250 K Hence, FTS – FRQ = 0
m
'
1.44  105
Therefore the force will act only on the segment SR whose
Example - 50 value is Bil and it’s direction is +z.
A solenoid has a core of a material with relative permeability
Example - 52
400. The windings of the solenoid are insulated from the
core and carry a current of 2 A. If the number of turns is An electron moves straight inside a charged parallel plate
1000 per metre, calculate (i) H (ii) B (iii) Intensity of c a p a c i t o r a . The space between
t u n i f o r m c h a r g e d e n s i t y

magnetisation I, and the magnetising current. the plates is filled with constant magnetic field of induction

B. Time of straight line motion of the electron in the
Sol. Here, r = 400, I’ = 2A, n = 1000 per metre
capacitor is
(i) H = nI’ = 1000 × 2 = 2 × 103 Am–1
(ii) B = H = 0 r H = 4× 10–7 × 400 (2 × 103) = 1.0 T
(iii) From B = 0 (H + I), where I is intensity of magnetisation, × × × × × × ×
× – × × × × × ×
e
B 1.0 × × × × × × ×
I H   2  103
0 4 107 × × × × × × ×

= 7.95 × 105 – 0.02 × 105 = 7.93 × 105 Am–1.


(iv) The magnetising current Im is the additional current that
needs to be passed through the windings of the solenoid in e  0 B
the absence of the core, which would produce a B value as (a)  B (b)
0 
in the presence of the core. Thus,
B = 0 n (I’ + Im) e 0B
(c)  B (d) e 
1.0 = 4 × 10–7 × 1000 (2 + Im) 0

1.0
Im   2  796  2  794 A Sol. (b)
4 104
The net force acting on the electron is zero because it moves
Example - 51 with constant velocity, due to it’s motion on straight line.
A conductor PQRSTU, each side of length L, bent as shown     
 Fnet  Fe  Fm  0  | Fe |  | Fm |
in the figure, carries a current i and is placed in a uniform
magnetic induction B directed parallel to the positive Y-axis.  eE = evB
The force experience by the wire and its direction are
E   
Z
R  ve   E  
B B 0B   0
i
S Q P   0 B
Y  The time of motion inside the capacitor . t   .
v 
T U
X

Mahesh Tutorials Science


MAGNETISM 327

Example - 53 Sol. (d)


A proton of mass m and charge +e is moving in a circular N
orbit of a magnetic field with energy 1MeV. What should 0ni; where n 
2R
B =

be the energy of -particle (mass = 4 m and charge = +2e),


so that it can revolve in the path of same radius 500
 B  4  10  7   0.5  5  10  4 T.
(a) 1 MeV (b) 4 MeV 2   0 .1
(c) 2 MeV (d) 0.5 MeV
Example - 56
Sol. (a) Figure shows a square loop ABCD with edge length a.
The resistance of the wire ABC is r and that of ADC is 2r.
2 mK
By using r  ; r  same, B  same The value of magnetic field at the centre of the loop
qB
assuming uniform wire is
q2 B
 K
m
2 2
K  q  m p  2q p  m
Hence          p 1= 1
K p  q p  m   q p  4m p
 A C
 K = Kp = 1meV. O
i

Example - 54
For the solenoid shown in figure. The magnetic field at
point P is D
n turn
2  0i 2  0i
(a) (b) 
30° 60° 3 a 3 a
P
2  0i 2  0i
(c) (d) 
(a)
 0 ni
4
 3  1 (b)
3  0 ni
4
a a

Sol. (b)
(c)
 0 ni
2
 3  1 (d)
 0 ni
4
 3  1 According to question resistance of wire ADC is twice that
of wire ABC. Hence current flows through ADC is half
Sol. (a)
i2 1
that of ABC i.e.  . Also i1 + i2 = 1
0 i1 2
B . 2 ni sin   sin  .
4
2i i
From figure  = (90o – 30o) = 60o and  = (90o – 60o) = 30o  i1  and i 2 
3 3

 B
 0 ni
2
sin 60  sin 30   0 ni
4
 3  1. Magnetic field at centre O due to wire AB and BC

 0 2i1 sin 45


Example - 55 (part 1 & 2) B1  . 
4 /2
The average radius of a toroid made on a ring of non-
magnetic material is 0.1 m and it has 500 turns. If it carries  0 2 2 i1 and magnetic field at centre O due to wires
 . 
0.5 ampere current, then the magnetic field produced along 4 
its circular axis inside the toroid will be AD and DC
(a) 25 × 10–2 Tesla (b) 5 × 10–2 Tesla
0 2 2 i2
(i.e. part 3 and 4) B3  B 4 
(c) 25 × 10–4 Tesla (d) 5 × 10–4 Tesla 4 

Lakshya Educare
328 MAGNETISM

Also i1 = 2i2. So (B1 = B2) > (B3 = B4) Square coil


i
Hence net magnetic field at centre O

Bnet = (B1 + B2) – (B3 + B4)


i
45°
O
45°

B i

a/2
(1) (2)
Length L = 4a
i1
A C
0 2 2 i 0 8 2 i
O B . B net  4B  .
i i2 4 a 4 a
(3) (4) B circular 2
Hence B 
square 8 2
D
Example - 58
What is the net force on the coil
2  i
2 2  i 2 2  2 10 cm
  3   0 . 3
 2 0 . 2A
4 a 4 a 1A
15 cm
2 cm
0 4 2 i
 . 2  1  2  0i 
4 3 a 3a
(a) 25 × 10–7 N moving towards wire
Example - 57 (b) 25 × 10–7 N moving away from wire
The ratio of the magnetic field at the centre of a current (c) 35 × 10–7 N moving towards wire
carrying circular wire and the magnetic field at the centre
(d) 35 × 10–7 N moving away from wire
of a square coil made from the same length of wire will be
Sol. (a)
2 2
(a) (b) Force on sides BC and CD cancel each other.
4 2 8 2
Force on side AB
 
(c) (d) 2  2 1
2 2 4 2 FAB  10  7   15  10  2  3  10  6 N
2  10  2
Sol. (b) Force on side CD
Circular coil
2  2 1
i FAB  10  7   15  10  2  0.5  10  6 N
12  10  2

r
B 10 cm C
2A 1A 15 cm
FAB FCD
i 2 cm
Length L = 2 r A D
Hence net force on loop = FAB – FCD = 25  10–7 N (towards
 2i  0 42i
Magnetic field B  0 .  . the wire).
4 r 4 r

Mahesh Tutorials Science


MAGNETISM 329

EXERCISE - 1 : BASIC OBJECTIVE QUESTIONS

Magnetic Field due to point charge 6. Equal current I flows in two segments of a circular loop in
the direction shown in figure
1. A moving charge will produce
(a) no field (b) an electric field
(c) a magnetic field (d) both ‘b’ and ‘c’
Magnetic Field due to Current

2. An element d   dxiˆ (where dx = 1 cm) is placed at the
origin and carries a large current I = 10A. What is the
magnetic field on the y-axis at a distance of 0.5 m ?

(a) 2  108 kT
ˆ (b) 4 108 kT
ˆ Radius of the loop is r. The magnitude of magnetic field
induction at the centre of the loop is
(c) 2  108 kT
ˆ (d) 4  108 kT
ˆ
 0 i
(a) zero (b)
Right hand rule 4r
3. A current carrying power line carries current from west to
0 i 0 i
east. The direction of magnetic field 1m above the power (c)     (d)  2   
line will be 2 r 2 r

(a) east to west (b) west to east 7. Ratio of magnetic field at the centre of a current carrying
coil of radius R and a distance 3R on its axis is
(c) south to north (d) north to south
Current loop (a) 10 10 (b) 20 10
4. A circular coil A of radius r carries current I. Another circular
(c) 30 10 (d) 5 10
coil B of radius 2r carries current of I. The magnetic fields at
the centres of the circular coils are in the ratio of 8. Three rings each having equal radius R are placed mutually
(a) 3 : 1 (b) 4 : 1 perpendicular to each other and each having centre at the
origin of coordinate axes system .If current I is flowing
(c) 1 : 1 (d) 2 : 1
through each ring then the magnitude of the magnetic field
5. A circular conducting ring of radius R is connected to two at the common centre is
exterior straight wires ending at two ends of a diameter.
The current I split into unequal portions while passing z axis
through the ring as shown. What is magnetic field induction
at the centre of the ring?

I/4 x axis
O
I R I
y axis
3I/4

I I (a)
3 0 I
(b)
 
3  1 0 I
(a) 0 (b) 0 2R 2R
4R 8R

0 I (c)
 
3  2 0 I
(d)
 
2  1 0 I
(c) (d) zero 2R 2R
3R

Lakshya Educare
330 MAGNETISM

9. A coil of 50 turns and 10 cm diameter has resistance of 10 14. The magnetic flux density B at a distance r from a long
ohm. What must be potential difference across the coil straight rod carrying a steady current varies with r as shown
so as to nullify the earth’s magnetic field B = 0.314 G at in figure
the centre of the coil.
B B
(a) 0.5 volt (b) 1.0 volt
(c) 1.5 volt (d) 2.5 volt
Straight Current Wire
10. Two very long straight parallel wires carry currents I and 2I (a) (b)
in opposite directions. The distance between the wires is r.
At a certain instant of time a point charge q is at a point O r O r
equidistant from the two wires in the plane of the wires. Its
instantaneous velocity v is perpendicular to this plane. B B
The magnitude of the force due to the magnetic field acting
on the charge at this instant is

30 Iqv (c) (d)


(a) zero (b)
2 r

 0 Iqv  0 Iqv O r O r
(c) (d)
 r 2 r
15. A current of i ampere flows along an infinitely long straight
11. The magnetic field at the point of intersection of the thin walled tube, then the magnetic induction at any point
diagonals of a square loop of side length L carrying inside the tube is
current I is
(a) infinite (b) zero
2 20 I 2 0 I
(a) (b) 0 2i 0i0
L L (c)  tesla  (d) tesla
4r 2r

2 0 I 4 20 I Solenoid
(c) (d)
L L 16. A solenoid of 1.5 metre length and 4.0 cm diameter possesses
12. A current I flowing through the sides of an equilateral 10 turn per cm. A current of 5 ampere is flowing through it.
triangle of side a. The magnitude of the magnetic field at The magnetic induction at axis inside the solenoid is
the centroid of the triangle is (a) 2 × 10–3 tesla (b) 2 × 10–5 tesla

2 0 I 3 3 0 I (c) 2 × 10–2 gauss (d) 2× 10–5 gauss


(a) (b)
a 2a 17. At the mid point along the length of a long solenoid, the
magnetic field is equal to B. If the length of solenoid is
9 0 I 2 20 I doubled and the current is reduced to half, the magnetic
(c) (d)
2a 2a field at the new mid point will nearest to
Inside and outside wire (a) 2B (b) B
13. A long, straight, solid metal wire of radius 2 mm carries a (c) B/4 (d) B/2
current uniformly distributed over its circular cross-section.
18. A long solenoid is formed by winding 20 turns/cm. The
The magnetic field induction at a distance 2 mm from its axis
current necessary to produce a magnetic field of 20 milli
is B. Then the magnetic field induction at distance 1 mm
from axis will be tesla inside the solenoid will be aproximately

(a) B (b) B/2 (a) 1.0 A (b) 2.0 A


(c) 2B (d) 4B (c) 4.0 A (d) 8.0 A

Mahesh Tutorials Science


MAGNETISM 331

19. A long solenoid has 800 turns per metre length of solenoid. 
25. A uniform magnetic field B  B0ˆj exists in space. A particle
A current of 1.6 A flows through it. The magnetic induction
at the end of the solenoid on its axis is of mass m and charge q is projected towards x-axis with
speed v from a point (a, 0, 0). The maximum value of v for
(a) 16 × 10–4 tesla (b) 8 × 10–4 tesla which the particle does not hit the y-z plane is
(c) 32 × 10–4 tesla (d) 4 × 10–4 tesla
Bqa Bqa
20. A toroidal solenoid has 3000 turns and a mean radius of (a) (b)
m 2m
10 cm. It has soft iron core of relative permeability 2000.
What is the magnitude of magnetic field in the core when Bq Bq
a current of 1 A is passed through the solenoid. (c) (d)
am 2am
(a) 1.2 T (b) 12 T 26. A charge +q is moving upwards vertically. It enters a
(c) 5.6 T (d) 4.5 T magnetic field directed to the north. The force on the charged
Magnetic field will be towards
21. A magnetic field (a) north (b) south
(a) always exerts a force on a charged particle (c) west (d) east
(b) never exerts a force on a charged particle 27. An electron has a circular path of radius 0.01 m in a
perpendicular magnetic induction 10–3 T. The speed of the
(c) exerts a force, if the charged particle is moving across
electron is nearly
the magnetic field lines
(a) 1.76 × 104 m/s (b) 1.76 × 106 m/s
(d) exerts a force, if the charged particle is moving along the
magnetic field lines (c) 3.52 × 106 m/s (d) 7.04 × 106 m/s
Motion Circular 28. A charged particle enters a uniform magnetic field with
velocity vector at an angle of 45º with the magnetic field. The
22. Imagine that you are seated in a room and there is a uniform
pitch of the helical path is p. The radius of the helix will be
magnetic field pointing vertically downwards. At the center
of the room, an electron is projected horizontally with a p p
certain speed. Discuss the speed and the path of the electron (a) (b)
 2
in this field.
(a) electron moves in anticlockwise path p
(c) 2p (d)
(b) electron moves in clockwise path 2 
(c) electron moves left wards 29. A deutron of kinetic energy 50 keV is describing a circular
(d) electron moves right wards orbit of radius 0.5 metre in a plane perpendicular to magnetic

23. A charged particle moving in a uniform magnetic field field B . The kinetic energy of the proton that describes a
penetrates a layer of lead and thereby loses one half of its circular orbit of radius 0.5 metre in the same plane with the
kinetic energy. How does the radius of curvature of its path 
same B is
change ?
(a) 200 keV (b) 100 keV
(a) The radius increases to r 2
(c) 50 keV (d) 25 keV
(b) The radius reduces to r / 2 Lorentz force
(c) The radius remains the same 30. An electron and a proton travel with equal speed in the
same direction at 90º to a uniform magnetic field as this is
(d) The radius becomes r/2
switched on. They experience forces which are initially
24. If a charged particle is describing a circle of radius r in a
magnetic field with a time period T, then (a) identical
(b) equal but in opposite direction
(a) T 2  r 3 (b) T 2  r
(c) in the same direction but differing by a factor of about 1840
(c) T  r 2 (d) T  r 0 (d) in opposite direction and differing by a factor of about 1840

Lakshya Educare
332 MAGNETISM

31. The mass of a proton is 1840 times that of electron. If an Force on straight current wire
electron and a proton are injected in a uniform electric field 36. The current in wire is directed towards east and the wire is
at right angle to the direction of the field, with the same placed in magnetic field directed towards north. The force
kinetic energy, then on the wire is
(a) the proton trajectory will be less curved than that of (a) vertically upwards (b) vertically downwards
electron
(c) due south (d) due east
(b) both the trajectories will be straight
37. A current of 3 A is flowing in a linear conductor having a
(c) both the trajectories will be equally curved length of 40 cm. The conductor is placed in a magnetic field
(d) the electron trajectory will be less curved than that of of strength 500 gauss and makes an angle of 30° with the
proton direction of the field. It experiences a force of magnitude
32. An electron is moving along positive x axis. A uniform (a) 3 × 10–4 N (b) 3 × 10–2 N
electric field exists towards negative y axis. What should (c) 3 × 102 N (d) 3 × 104 N
be the directions of the magnetic field of suitable
38. A charged particle is whirled in a horizontal circle on a
magnitude so that net force on the electron is zero?
frictionless table by attaching it to a string fixed at one
(a) positive y axis (b) positive z axis point. If the magnetic field is switched on in the vertical
(c) negative z axis (d) negative y axis. direction the tension in the string

Parallel Fields (a) will increase

33. A uniform electric field and a uniform magnetic field are (b) will decrease
pointed in the same direction. If an electron is projected in (c) may increase or decrease
the same direction, the electron (d) will remain unchanged
(a) velocity will increase in magnitude 39. A current of 10 ampere is flowing in a wire of length 1.5
(b) velocity will decrease in magnitude metre. A force of 15 newtons acts on it when it is placed in a
(c) will turn to its left uniform magnetic field of 2 tesla. The angle between the
magnetic field and the direction of the current is
(d) will turn to its right
(a) 30° (b) 45°
Under uniform magnetic field
(c) 60° (d) 90°
34. A metal wire of mass m slides without friction on two rails
40. A current I1 carrying wire AB is placed near an another long
placed at a distance  apart. The track lies in a uniform vertical
wire CD carrying current I2. Figure. If free to move, wire AB
magnetic field B. A constant current I flows along the rails
will have
across the wire and back down the other rail. The acceleration
of the wire is

BmI
(a) (b) mBI

BI mI
(c) (d)
m B
35. A straight horizontal wire of mass 10 mg and length 1 m
carries a current of 2 ampere .What minimum magnetic field
B should be applied in the region so that the magnetic (a) rotational motion only
force on the wire may balance its weight. (b) translational motion only
-4 -4
(a) 2.45 × 10 T (b) 4.9 × 10 T (c) rotational as well as translational motion
(c) 4.9 × 10-5 T (d) 9.8 × 10-4 T (d) neither rotational nor translational motion

Mahesh Tutorials Science


MAGNETISM 333

Torque on Current loop 46. A current of 2 ampere is passed in a coil of radius 0.5 m and
41. Four wire loops each of length 2.0 metres are bent into four number of turns 20. The magnetic moment of the coil is
loops P, Q, R and S and then suspended in a uniform (a) 0.314 Am2 (b) 3.14 A–m2
magnetic field. Same current is passed in each loop. Which (c) 314 A–m2 (d) 31.4 A–m2
statement is correct ?
47. The area of cross-section of three magnets of same length
are A, 2A and 6A respectively. The ratio of their magnetic
moments will be
(a) 6 : 2 : 1 (b) 1 : 2 : 6
(c) 1 : 4 : 36 (d) 36 : 4 : 1
48. Magnetic field at the centre of the circular loop of area A
is B. Then the magnetic moment of the loop will be
(a) Couple on loop P will be the highest
(b) Couple on loop Q will be the highest BA A BA A
(a) 0 (b) 0
(c) Couple on loop R will be the highest 2 
(d) Couple on loop S will be the highest
42. A circular loop of area 1 cm2 carrying a current of 10 ampere 2BA A
(c)  (d) none of these
is placed in a magnetic field of 0.1 T perpendicular to plane 0 
of the loop.The torque on the loop due to magnetic field is
49. A magnetic needle has magnetic moment of 6.7×10-2 A.m2
(a) 10-4 N.m (b) 10-2 N.m
and moment of inertia 7.5 × 10-6 kgm2.It performs 10 com-
(c) 10 N.m (d) zero plete oscillations in 6.7 seconds .What is the magnitude of
43. A wire of length  in formed into a circular loop of one turn the magnetic field.
only and is suspended in a magnetic field B. When a current (a) 0.01 T (b) 0.2 T
I is passed through the loop, the torque experienced by it is (c) 0.5 T (d) 0.9T
2
(a) (1/4)BI (b) (1/4) IB Current Sensitivity, Voltage
2
(c) (1/4)B I (d) (1/4)BI  2
50. The sensitivity of a galvanometer does not depend upon
44. A conducting ring of mass 2 kg and radius of 0.5 m is placed (a) a very strong magnetic field in the permanent magnet
on a smooth horizontal plane .The ring carries a current of
4 A. A horizontal magnetic field B=10 T is switched on at (b) the current it measures
t=0 as shown in diagram. What is initial angular acceleration (c) a very thin, weak suspension
of the ring ?
(d) a large number of turns in the coil
I Between Parallel Currents
51. The forces existing between two parallel current carrying
R
B conductors is F. If the current in each conductor is doubled,
then the value of force will be
(a) 2F (b) 4F
(c) 5F (d) F/2
(a) 40π rad/s (b) 20π rad/s 52. Two parallel wires carry currents of 20 A and 40 A in opposite
(c) 10 π rad/s (d) zero directions. Another wire carrying current of 20 A and anti-
Magnetic moment parallel to 20A is placed midway between the two wires
.The magnetic force on this wire will be
45. A wire of length L metre carrying a current I ampere is bent
in the form of a circle. Its magnitude of magnetic moment (a) towards 20 A
will be (b) towards 40 A
(a) IL/4 (b) I2L2/4 (c) perpendicular to plane of wires
(c) I2L/8 (d) IL2/4 (d) zero

Lakshya Educare
334 MAGNETISM

53. Through two parallel wires A and B, 10A and 2A of currents Magnetic Moment
are passed respectively in opposite directions. If the wire A 59. A steel wire of length  has a magnetic moment M. It is bent
is infinitely long and the length of the wire B is 2m, then into L shape from the middle. The new magnetic moment is
force on the conductor B, which is situated at 10 cm distance
from A, will be (a) M (b) M / 2

(a) 8 × 10–7 N (b) 8 × 10–5 N (c) M/2 (d) 2M


–7 –5
(c) 4 × 10 N (d) 4 × 10 N Magnetic Field
54. If a current is passed in a spring, it 60. A bar magnet of length 3 cm has a point A and B along axis
(a) gets compressed at a distance of 24 cm and 48 cm on the opposite ends. Ratio
(b) gets expanded of magnetic fields at these points will be

(c) oscillates
(d) remains unchanged
55. Choose the correct statement. There will be no force
(a) 8 (b) 3
experienced if
(a) Two parallel wires carry current in same direction (c) 4 (d) 1/ 2 2

(b) A positive charge is projected along the axis of the 61. A short bar magnet of length 4 cm has a magnetic moment
solenoid 1
of 4JT .What is the magnitude of the magnetic field at a
(c) A positive charge is projected between the pole pieces
distance 2 m from the centre of the magnet on its equatorial
of a bar magnet
line.
(d) Two protons move parallel to each other with same
speed (a) 2 105 T (b) 5 × 10–8 T

Conversion (c) 1.2  107 T (d) 3.4  105 T


56. The deflection in a galvanometer falls from 50 division to
62. What is the magnetic field due to a dipole of magnetic
20 when a 12 ohm shunt is applied. The galvanometer
moment 1.2Am2 at a point 1 m away from it .The observation
resistance is
point is in a direction making an angle of 60o with the
(a) 18 ohms (b) 36 ohms dipole axis.
(c) 24 ohms (d) 30 ohms (a) 1.6 × 10–7 T (b) 1.2 × 10–6 T
57. A galvanometer of resistance 100  gives a full scale (c) 1.2 × 10–4 T (d) 1.73 × 10–5 T
deflection for a current of 10–5 A. To convert it into a ammeter
63. Two identical dipoles each of magnetic moment 1 A m2 are
capable of measuring upto 1 A, we should connect a
placed at a separation of 2 m with their axes perpendicular
resistance of
to each other. What is the magnetic field at a point midway
(a) 1 in parallel between the dipoles ?
(b) 10–3  in parallel
(c) 105  in series
(d) 100  in series 2m M2
M1
58. We have a galvanometer of resistance 25 . It is shunted by
a 2.5  wire. The part of total current I0 that flows through
the galvanometer is given as (a) 1.2×10-4 T (b) 3.1×10-3T
(a) (I/I0) = (1/11) (b) (I/I0) = (1/10)
(c) 7.6×10-5T (d) 5×10-7 T
(c) (I/I0) = (1/9) (d) (I/I0) = (2/11)

Mahesh Tutorials Science


MAGNETISM 335

64. Of the following figure, the lines of magnetic induction due 69. The angles of dip at the poles and the equator respectively are
to a magnet SN, are given by (a) 30°, 60° (b) 90°, 0°
(c) 30°, 90° (d) 0°, 0°
70. At a certain place, the horizontal component of the earth’s
magnetic field is B0 and the angle of dip is 45°. The total
intensity of the field at that place will be
(1) (2)
(a) B0 (b) 2 B0

(c) 2B0 (d) B02

71. At a certain place on earth, a magnetic needle is placed


along the magnetic meridian at an angle of 60° to the
horizontal. If the horizontal component of the magnetic
field at the place is found to be 2 105 T . What is the
magnitude of total earth’s field at that place.
(3) (4)
(a) 2 104 T (b) 4 105 T

(c) 105 T (d) 3  10 5 T


72. Agonic line is that curve at which
(a) 1 (b) 2 (a) total intensity of earth ‘s magnetic field is same
(c) 3 (d) 4 (b) the angle of dip is same
65. A thin rectangular bar magnet suspended freely has period (c) angle of declination is same
of oscillation of 4 seconds. What will be period of oscillation
(d) magnetic declination is zero
if the magnet is broken into two halves; each having length
half of original; and one piece is made to oscillate in the 73. The magnetic lines of force due to horizontal component of
same field. earth’s magnetic field will be
(a) 2 s (b) 3 s (a) elliptical
(c) 1 s (d) 4 s (b) circular
Earth Magnetism (c) horizontal and parallel
66. The total intensity of the Earth’s magnetic field at equator is (d) curved
5 units. What is its value at the poles ? 74. The magnetic induction along the axis of an air cored
(a) 5 (b) 4 solenoid is 0.03 T. On placing an iron core inside the
(c) 3 (d) 2 solenoid the magnetic induction becomes 1.5T .The relative
permeability of iron core will be
67. At a certain place, horizontal component of Earth’s field is
(a) 12 (b) 40
3 times the vertical component. The angle of dip at this
(c) 50 (d) 300
place is
Magnetizing Field Intensity
(a) 0 (b) /3
75. An iron rod of length 20 cm and diameter 1 cm is placed
(c) /6 (d) none of the above
inside a solenoid on which the number of turns is 600. The
68. In a magnetic meridian of a certain place, horizontal compo- relative permeability of the rod is 1000. If a current of 0.5 A is
nent of earth’s field is 0.25G and the angle of dip is 60o.What placed in the solenoid, then the magnetisation of the rod
is the magnetic field of the earth at this location. will be
(a) 0.5G (b) 0.25 G (a) 2.997 × 102 A/m (b) 2.997 × 103 A/m
(c) 0. 25 3 G (d) none of these (c) 2.997 × 104 A/m (d) 2.997 × 105 A/m

Lakshya Educare
336 MAGNETISM

76. The mass of iron rod is 80 gm and its magnetic moment is (a) 1000 A/m (b) 1400 A/m
10A.m2.If the density of iron is 8 gm/cc, then the value of (c) 2000 A/m (d) 2400A/m
intensity of magnetization will be
83. In the above problem, magnetizing field in the presence
(a) 106A/m (b) 3000 A/m
of core will be
(c) 105 A/m (d) 1A/m
(a) 1000 A/m (b) 2000 A/m
77. A solenoid has core of a material with relative permeabil-
ity 400.The winding of the solenoid are insulated from (c) 2400 A/m (d) 3200 A/m
the core and carry a current of 2 ampere. If the number of 84. The intensity of magnetization in the presence of core
turns is 1000 per meter, what is magnetic flux density in- will be
side the core?
(a) 1000 A/m (b) 2.3 × 104 A/m
(a) 0.4T (b) 0.5 T
(c) 0.7 T (d) 1.0T (c) 7.94 × 105 A/m (d) 4.3 × 10-5 A/m
Susceptibility 85. The magnetization in the absence of the core will be
78. The magnetic susceptibility of a material of a rod is 499. (a) 2400 A/m (b) 2.3 × 104 A/m
Permeability of vacuum is 4 × 10 –7 H/m. Absolute
permeability of the material of the rod in henry/meter is (c) 7.94 × 105 A/m (d) zero

(a) × 10–4 (b) 4 × 10–4 86. The relative permeability of the material will be

(c) 3 × 10–4 (d) 2 × 10–4 (a) 397.7 (b) 448.5


79. Magnetic susceptibility is negative for (c) 533 (d) 657
(a) Paramagnetic material only 87. The coercivity of a bar magnet is 4000A/m .In order to
(b) Diamagnetic material only demagnetize it is placed inside a solenoid of length 12 cm
(c) Ferromagnetic material only and having 60 turns. What current should be passed
(d) Paramagnetic and Ferromagnetic materials through the solenoid?
Magnetic permeability (a) 2A (b) 4A
80. A magnetising field of 2 × 103 amp/m produces a magnetic (c) 8A (d) 16A
flux density of 8 tesla in an iron rod. The relative Ferromagnetic
permeability of the rod will be
88. A uniform magnetic field parallel to the plane of paper,
(a) 102 (b) 100
existed in space initially directed from left to right. When a
(c) 103 (d) 104 bar of soft iron is placed in the field parallel to it, the lines
81. The main difference between electric lines of force and of force passing through it will be represented by figure
magnetic lines of force is
(a) Electric lines of force are closed curves whereas
magnetic lines are open curve
(b) Electric lines of force are open curve and magnetic (A) (B)
lines are closed curve
(c) Magnetic field lines cut each other whereas
electric lines don’t
(d) Electric lines of force cut each other whereas magnetic
lines of force don’t cut
82. There are 1000 turns /m in a Rowland’s ring and a current (C) (D)
of 2A is flowing in the windings .The value of magnetic
induction produced is found to be 1.0T.When no core is
present then magnetizing field produced in the ring will (a) A (b) B
be (c) C (d) D

Mahesh Tutorials Science


MAGNETISM 337

89. A sensitive magnetic instrument can be shielded very Curies Law


effectively from outside magnetic field by placing it 94. Curie’s law states that
inside a box of (a) magnetic susceptibility is inversely proportional to the
(a) Teak wood absolute temperature
(b) plastic material (b) magnetic susceptibility is inversely proportional the
square root of the absolute temperature
(c) A metal of low magnetic permeability
(c) magnetic susceptibility is directly proportional to the
(d) A metal of high magnetic permeability absolute temperature
90. When a Ferromagnetic substance is heated to a (d) magnetic susceptibility does not depend on temperature
temperature above its Curie temperature it Hysteris Curve
(a) behaves like Diamagnetic material 95. The hysterisis curve is studied generally for
(b) behaves like Paramagnetic material (a) ferromagnetic materials
(c) is permanently demagnetized (b) paramagnetic materials
(d) remains Ferromagnetic (c) diamagnetic materials

91. A Ferromagnetic material is placed in an external magnetic (d) all of these


field. The magnetic domains 96. The B–H curve (i) and (ii) shown in figure associated with
(a) increase in size
(b) decrease in size
(c) may increase or decrease in size
(d) have no relation with field
Diamagnetic
92. For a diamagnetic material

(a)  r  1,  m  1 (b)  r  1,  m  1
(a) (i) diamagnetic and (ii) paramagnetic substance
(c)  r  1,  m  0 (d)  r  1,  m  0 (b) (i) paramagnetic and (ii) ferromagnetic substance
(c) (i) Soft iron and (ii) Steel respectively
93. Water is
(d) (i) steel and (ii) Soft iron respectively
(a) diamagnetic (b) paramagnetic
Permanent Magnets
(c) ferromagnetic (d) none of these
97. The most suitable metal for permanent magnet is
(a) copper (b) aluminium
(c) steel (d) iron

Lakshya Educare
338 MAGNETISM

EXERCISE - 2 : PREVIOUS YEAR COMPETITION QUESTIONS


PREVIOUSYEARSAFMC QUESTIONS 7. Which one is a vector quantity ? (AFMC 2003)
1. A current carrying wire in the neighbourhood produces (a) Time (b) Temperature
(AFMC 1999)
(c) Flux density (d) Magnetic field intensity
(a) electric and magnetic fields
8. A wire carrying current I and other carrying 2 I in the same
(b) magnetic field only direction produce a magnetic field B at the mid-point. What
(c) no field will be the field when 2 I wire is switched off ?
(a) B/2 (b) 2B (AFMC 2005)
(d) electric field
(c) B (d) 4B
When a charged particle moving with velocity v is
2. Two parallel wires in free space are 10 cm apart and each
9.
carries a current of 10A in the same direction. The force 
subjected to a magnetic feild of induction B , the force on
exerted by one wire on other per metre of length of the
it is non-zero. This implies that (AFMC 2006)
wire is (AFMC 1999)  
–6 –4 (a) angle between v and B is necessarily 90°
(a) 2 × 10 N (b) 2 × 10 N
 
–3 –2 (b) angle between v and B can have any value other
(c) 2 × 10 N (d) 2 × 10 N than 90°
3. A long hollow copper pipe carries a current, then magnetic  
(c) angle between v and B can have any value other
field produced is (AFMC 1999) than zero and 180°
 
(a) both inside and outside the pipe (d) angle between v and B is either zero or 180°
(b) neither inside nor outside the pipe 10. The path of an electron in a uniform magnetic field may be
(c) outside the pipe only (a) circular but not helical (AFMC 2007)
(b) helical but not circular
(d) inside the pipe only
(c) neither helical nor circular
2
4. A coil having 100 turns and area of 0.001 m is free to (d) either helical or circular
rotate about an axis, the coil is placed perpendicular to a 11. A straight wire of mass 200 g and length 1.5 m carries a
2
magnetic field of 1.0 Wb/m . If the coil is rotated rapidly current of 2 A. It is suspended in mid-air by a uniform
through an angle of 180°, how much charge will flow horizontal magnetic field B. The magnitude of B (in tesla)
–2
through the coil ? The resistance of the coil is 10. is (assume g = 9.8 ms ) (AFMC 2008)
(a) 2 (b) 1.5
(AFMC 2002)
(c) 0.55 (d) 0.65
(a) 0.02 C (b) 0.04 C
12. A wire PQR is bent as shown in fig.and is placed in a region
(c) 0.08 C (d) 0.01 C of uniform magnetic field B. The length of PQ = QR = l. A
2 current I ampere flows through the wire as shown. The
5. Wb/m is equal to (AFMC 2001)
magnitude of the force on PQ and QR will be
(a) dyne (b) tesla (AFMC 2008)
(c) watt (d) henry R

6. Two wires carry current in different directions. They will


(AFMC 2003) B
I
(a) attract each other
(b) repel each other
P Q
I
(c) create gravitational field
(a) BIl, 0 (b) 2BIl, 0
(d) any of the above
(c) 0, BIl (d) 0, 0

Mahesh Tutorials Science


MAGNETISM 339

13. When a positively charged particle enters a uniform 20. A uniform magnetic field acts right angles to the direction
magnetic field with uniform velocity, its trajectory can be of motion of electrons. As a result, the electron moves in a
(AFMC 2008) circular path of radius 2 cm. If the speed of electrons is
doubled, then the radius of the circular path will be
(1) a straight line (2) a circle
(CBSE–PMT 1991)
(3) a helix
(a) 2.0 cm (b) 0.5 cm
(a) (1) only (b) (1) or (2) (c) 4.0 cm (d) 1.0 cm
(c) (1) or (3) (d) any one of (1), (2) and (3) 21. The magnetic field at a distance ‘r’ from a long wire
carrying current ‘i’ is 0.4 Tesla. The magnetic field at a
PREVIOUS YEARS CBSE–PMT QUESTIONS
distance ‘2r’ is (CBSE–PMT 1992)
14. A current carrying coil is subjected to a uniform magnetic (a) 0.2 Tesla (b) 0.8 Tesla
field. The coil will orient so that its plane becomes
(c) 0.1 Tesla (d) 1.6 Tesla
(CBSE–PMT 1988)
22. A straight wire of length 0.5 metre and carrying a current
(a) inclined at 45° to the magnetic field
of 1.2 ampere is placed in uniform magnetic field of
(b) inclined at any arbitrary angle to the magnetic field induction 2 Tesla. The magnetic field is perpendicular to
(c) parallel to the magnetic field the length of the wire. The force on the wire is
(CBSE–PMT 1992)
(d) perpendicular to magnetic field
(a) 2.4 N (b) 1.2 N
15. Tesla is the unit of (CBSE–PMT 1988) (c) 3.0 N (d) 2.0 N
(a) magnetic flux (b) magnetic field 23. To convert a galvanometer into an ammeter, one needs to
(c) magnetic induction (d) magnetic moment connect a (CBSE–PMT 1992)
16. Energy in a current carrying coil is stored in the form of (a) low resistance in parallel
(CBSE–PMT 1989) (b) high resistance in parallel
(a) electric field (b) magnetic field (c) low resistance in series
(d) high resistance in series
(c) dielectric strength (d) heat
24. A coil carrying electric current is placed in uniform magnetic
17. The total charge induced in a conducting loop when it is
field (CBSE–PMT 1993)
moved in magnetic field depends on (CBSE–PMT 1990)
(a) torque is formed
(a) the rate of change of magnetic flux
(b) e.m.f. is induced
(b) initial magnetic flux only (c) both (a) and (b) are correct
(c) the total change in magnetic flux (d) none of these
(d) final magnetic flux only 25. A charge moving with velocity v in X-direction is
18. The magnetic induction at a point P which is at the distance subjected to a field of magnetic induction in negative
–3 X-direction. As a result, the charge will
of 4 cm from a long current carrying wire is 10 T. The
field of induction at a distance 12 cm from the current will (CBSE–PMT 1993)
be (CBSE–PMT 1990) (a) remain unaffected
(a) 3.33 × 10 T
–4
(b) 1.11 × 10 T
–4 (b) start moving in a circular path Y-Z plane
–3 –3 (c) retard along X-axis
(c) 3 × 10 T (d) 9 × 10 T
(d) moving along a helical path around X-axis
19. A deuteron of kinetic energy 50 keV is describing a circular
26. A electron enters a region where magnetic (B) and electric
orbit of radius 0.5 metre in a plane perpendicular to magnetic
(E) fields are mutually perpendicular, then
field B. The kinetic energy of the proton that describes a
(CBSE–PMT 1994)
circular orbit of radius 0.5 metre in the same plane with the
(a) it will always move in the direction of B
same B is (CBSE–PMT 1991)
(b) it will always move in the direction of E
(a) 25 keV (b) 50 keV
(c) it always possesses circular motion
(c) 200 keV (d) 100 keV
(d) it can go undeflected also

Lakshya Educare
340 MAGNETISM

27. A straight wire of diameter 0.5 mm carrying a current of 1A (c) both inside and outside the pipe
is replaced by another wire of 1mm diameter carrying same (d) no where
current. The strength of magnetic field far away is
34. A straight wire of diameter 0.5 mm carrying a current of 1A
(CBSE–PMT 1995) is replaced by another wire of diameter 1 mm carrying the
(a) twice the earlier value same current. The strength of magnetic field far away is
(b) same as the earlier value (CBSE–PMT 1999)
(c) one-half of the earlier value (a) twice the earlier value
(d) one-quarter of the earlier value (b) one-half of the earlier value
28. At what distance from a long straight wire carrying a (c) one quarter of the earlier value
–6
current of 12A will the magnetic field be equal to 3 × 10
2 (d) same as earlier value
Wb/m ? (CBSE–PMT 1995)
–1 –2 35. Magnetic field due to 0.1A current flowing through a
(a) 8 × 10 m (b) 12 × 10 m
circular coil of radius 0.1m and 1000 turns at the centre of
–2 –2
(c) 18 × 10 m (d) 24 × 10 m the coil is (CBSE–PMT 1999)
29. The magnetic field (dB) due to a small element (dl) at a –4
(a) 0.2 T (b) 2 × 10 T

distance ( r ) and element carrying current i is –8
(c) 4.9 × 10 T (d) 9.8 × 10 T
–4
(CBSE–PMT 1996) 3
36. An electron moves with a velocity 1 × 10 m/s in a magnetic
 
  dl  r  0 2  dl  r  field of induction 0.3 T at an angle 30°. If e/m of electron is
(a) dB  0 i   (b) dB  i   11
4  r  4  r 2  1.76 × 10 C/kg, the radius of the path is nearly
  (CBSE–PMT 2000)
  dl  r  0  dl  r 
(c) dB  0 i 2   (d) dB  i   –8
(a) 10 m (b) 2 × 10 m
–8
4  r  4  r 3 
–6 –10
30. A 10eV electron is circulating in a plane at right angles (c) 10 m (d) 10 m
–4 2 37. Current is flowing in a coil of area A and number of turns
to a uniform field at magnetic induction 10 Wb/m
(= 1.0 gauss). The orbital radius of the electron is N, then magnetic moment of the coil M is equal to
(CBSE–PMT 1996) (CBSE–PMT 2001)
(a) 12 cm (b) 16 cm (a) NiA (b) Ni/A
2
(c) Ni / A (d) N Ai
8 2
(c) 2 cm (d) 18 cm 38. A charged particle of charge q and mass m enters
 
31. Two parallel wires in free space are 10 cm apart, and each perpendicularly in a magnetic field B . Kinetic energy of
carries a current of 10A, in the same direction. The force, the particle is E; then frequency of rotation is
one wire exerts on the other, per metre of length, is (CBSE–PMT 2001)
(CBSE–PMT 1997) qB qB
(a) (b)
–7
(a) 2 × 10 N, repulsive
–7
(b) 2 × 10 N, attractive m 2m
–4 –4
(c) 2 × 10 N, repulsive (d) 2 × 10 N, attractive qBE qB
(c) (d)
32. For protecting a sensitive equipment from the external 2m 2E
magnetic field, it should be (CBSE–PMT 1998) 39. The magnetic field of a given length of wire carrying a
(a) placed inside an aluminium can current for a single turn circular coil at centre is B, then its
(b) placed inside an iron can value for two turns for the same wire when same current
passing through it is (CBSE–PMT 2002)
(c) wrapped with insulation around it when passing current
(a) B/4 (b) B/2
through it
(c) 2B (d) 4B
(d) surrounded with fine copper sheet 
40. A charge q moves in a region where electric field E and
33. If a long hollow copper pipe carries a current, then 
magnetic field is produced (CBSE–PMT 1999) magnetic field B both exist, then the force on it is
(CBSE–PMT 2002)
(a) inside the pipe only (b) outside the pipe only

Mahesh Tutorials Science


MAGNETISM 341

     (CBSE–PMT 2005)
(a) q v  B (b) q E  q v  B
   (a) B/v (b) v/B
  
(c) q B  q B  v 
(d) q B  q E  v  (c) v/B (d) B/ v
41. The magnetic flux through a circuit of resistance R changes
by an amount  in a time t. Then the total quantity of 46. When a charged particle moving with velocity v is
electric charge Q that passes any point in the circuit during 
subjected to a magnetic field of induction B , the force on
the time t is represented by (CBSE–PMT 2004)
it is non-zero. This implies that (CBSE–PMT 2006)
1   
(a) Q  . (b) Q  
R t R (a) angle between v and B is necessarily 90°

 
  (b) angle between v and B can have any value other than
(c) Q  (d) Q  R .
t t
90°
42. A coil in the shape of an equilateral triangle of side l is 

suspended between the pole pieces of a permanent magnet (c) angle between v and B can have any value other than

such that B is in plane of the coil. If due to a current i in zero and 180°
the triangle a torque  acts on it, the side l of the triangle is  
(CBSE–PMT 2005) (d) angle between v and B is either zero or 180°

2    2   
1/ 2
47. Under the influence of a uniform magnetic field a charged
(a)   (b)  
3  Bi  3  Bi  particle is moving in a circle of radius R with constant
speed v. The time period of the motion
1/ 2
   1  (CBSE–PMT 2007)
(c) 2   (d)
 3 Bi  3 Bi (a) depends on v and not on R
43. A very long straight wire carries a current I. At the instant (b) depends on both R and v
when a charge +Q at point P has velocity v , as shown, (c) is independent of both R and v
the force on the charge is (CBSE–PMT 2005) (d) depends on R and not on v
48. A charged particle (charge q) is moving in a circle of radius
R with uniform speed v. The associated magnetic moment
 is given by (CBSE–PMT 2007)
qvR 2
(a) (b) qvR
2
qvR 2
(c) (d) qvR
2
49. A closed loop PQRS carrying a current is placed in a uniform
(a) opposite to ox (b) along ox
magnetic field. If the magnetic forces on segments PS, SR
(c) opposite to oy (d) along oy and RQ are F1, F2 and F3 respectively and are in the plane
 
44. If the angle between the vectors A and B is , the value of the paper and along the directions shown, the force on
  
 
of the product B  A .A is equal to
the segment QP is (CBSE–PMT 2008)
Q
(CBSE–PMT 2005)
(a) BA cos 
2
(b) BA sin 
2 P
F3
(c) BA sin  cos 
2
(d) zero
F1
45. An electron moves in a circular orbit with a uniform speed
v. It produces a magnetic field B at the centre of the circle. S R
The radius of the circle is proportional to F2

Lakshya Educare
342 MAGNETISM

(b) putting in series a resistance of 15 


 F3  F1 
2
(a) F3 – F1 – F2 (b)  F22
(c) putting in series a resistance of 240 
 F3  F1  (d) putting in parallel a resistance of 15 
2
(c) F
2
2 (d) F3 – F1 + F2

50. A particle mass m, charge Q and kinetic energy T enters a 56. If a diamagnetic substance is brought near the north or
 the south pole of a bar magnet, it is (CBSE 2009)
transverse uniform magnetic field of induction B . After 3
s the kinetic energy of the particle will be (a) repelled by both the pole
(CBSE–PMT 2008)
(b) repelled by the north pole and attracted by the south
(a) 3T (b) 2T
pole
(c) T (d) 4T
(c) attracted by the north pole and repelled by the south
51. A circular disc of radius 0.2 m is placed in a uniform
pole
1  Wb 
magnetic field of induction   in such a way that (d) attracted by both the poles.
  m2 
 57. A bar magnet having a magnetic moment of 2 × 10 JT is
4 –1
its axis makes an angle of 60° with B . The magnetic flux
linked with the disc is (CBSE–PMT 2008) free to rotate in a horizontal plane. A horizontal magnetic
–4
field B = 6 × 10 T exists in the space. The work done in
(a) 0.02 Wb (b) 0.06 Wb
taking the magnet slowly from a direction parallel to the
(c) 0.08 Wb (d) 0.01 Wb
field to a direction 60° from the field is : (CBSE 2009)
52. A galvanometer of resistance 50  is connected to a battery
(a) 0.6 J (b) 12 J
of 3 V alongwith a resistance of 2950 in series. A full scale
deflection of 30 divisions is obtained in the galvanometer. (c) 6 J (d) 2 J
In order to reduce this deflection to 20 divisions, the 58. Charge q is uniformly spread on a thin ring of radius R.
resistance in series should be (CBSE 2008)
The ring rotates about its axis with a uniform frequency f
(a) 5050  (b) 5550  Hz. The magnitude of magnetic induction at the center of
(c) 6050  (d) 4450  the ring is : (CBSE 2011)
53. Under the influence of a uniform magnetic field, a charged
0 q 0 qf
particle moves with constant speed v in a circle of radius (a) (b)
R. The time period of rotation of the particle 2fR 2R

(CBSE 2009)
0 qf 0 q
(a) depends on v and not on R (c) (d)
2R 2fR
(b) depends on R and not on v
(c) is independent of both v and R 59. Two similar coils of radius R are lying concentrically with
their planes at right angles to each other. The currents
(d) depends on both v and R
flowing in them are I and 2I, respectively. The resultant
54. The magnetic force acting on a charged particle of charge
magnetic field induction at the centre will be(CBSE 2012)
– 2C in a magnetic field of 2 T acting in y direction, when

 
the particle velocity is 2iˆ  3jˆ × 10 ms is
6 –1
(a)
0 I
(b)
5 0 I
(CBSE 2009) R 2R
(a) 8 N in –z direction (b) 4 N in z direction
3 0 I 0 I
(c) 8 N in y direction (d) 8 N in z direction (c) (d)
2R 2R
55. A galvanometer having a coil resistance of 60  shown
full scale deflection when a current of 1.0 A passes through 60. A compass needle which is allowed to move in a horizontal
it.It can be converted into an ammeter to read currents plane is taken to a geomagnetic pole. It : (CBSE 2012)
upto 5.0 A by (CBSE 2009) (a) will stay in east-west direction only
(a) putting in parallel a resistance of 240 

Mahesh Tutorials Science


MAGNETISM 343

(b) will become rigid showing no movement 65. A bar magnet of length ‘’ and magnetic dipole moment
‘M’ is bent in the form of an arc as shown in figure. The
(c) will stay in any position
new magnetic dipole moment will be (CBSE 2013)
(d) will stay in north-south direction only
61. An alternating electric field, of frequency v, is applied
across the dees (radius = R) of a cyclotron that is being
used to accelerate protons (mass = m). The operating
magnetic field (B) used in the cyclotron and the kinetic

r
energy (K) of the proton beam, produced by it, are given 60°

by (CBSE 2012)

mv M
(a) B  and K  m 2 vR 2 (a) (b) M
e 2

3 2
mv (c) M (d) M
(b) B  and K  2m2 v 2 R 2  
e
PREVIOUSYEARSAIIMS QUESTIONS
2 mv 66.
–14
An electron moving with kinetic energy 6.6 × 10 J enters
(c) B  and K  m 2 vR 2
–3
e a magnetic field 4 × 10 T at right angle to it. The radius of
its circular path will be nearest to (AIIMS 1997)
2 mv
(d) B  and K  2m  2 v 2 R 2 (a) 100 cm (b) 75 cm
e
(c) 25 cm (d) 50 cm
62. A proton carrying 1 MeV kinetic energy is moving in a
67. Which one of the following statement is not correct about
circular path of radius R in uniform magnetic field. What
the magnetic field ? (AIIMS 2000)
should be the energy of an –particle to describe a circle
of same radius in the same field ? (CBSE 2012) (a) Inside the magnet the lines go from north pole to south
pole of the magnet
(a) 4 MeV (b) 2 MeV
(b) Tangents to the magnetic lines give the direction of
(c) 1 MeV (d) 0.5 MeV
the magnetic field
63. A magnetic needle suspended parallel to a magnetic field
(c) The magnetic lines form a closed loop
reqiures 3 J of work to turn it through 60°. The torque (d) Magnetic lines of force do not cut each other
needed to maintain the needle in this position will be
68. What should be amount of current through the ring of
(CBSE 2012) radius of 5 cm so that field at the centre is equal to the
–5 2
earth’s magnetic field 7 × 10 Wb/m is ? (AIIMS 2000)
3
(a) J (b) 2 3 J
2 (a) 0.28 A (b) 5.57 A
(c) 2.8 A (d) none of these
(c) 3 J (d) 3J
69. Which one of the following are used to express intensity
64. A current loop in a magnetic field : (CBSE 2013) of magnetic field in vacuum ? (AIIMS 2000)
(a) Can be in equilibrium in two orientations, one stable (a) Oersted (b) Tesla
while the other is unstable. (c) Gauss (d) None of these
(b) Experiences a torque whether the field is uniform or
non uniform in all orientations
(c) Can be in equilibrium in one orientation
(d) Can be in equilibrium in two orientations, both the
equilibrium states are unstable.

Lakshya Educare
344 MAGNETISM

70. An electron is travelling along the x-direction. It encounters the separation H is (AIIMS 2006)
a magnetic field in the y-direction. Its subsequent motion
R
will be (AIIMS 2003) Wire
Ic
(a) straight line along the x-direction
(b) a circle in the xz-plane H
(c) a circle in the yz-plane
(d) a circle in the xy-plane Ie
Straight
71. A rectangular loop carrying a current i1, is situated near a
long straight wire carrying a steady current i2. The wire is Ie R Ic R
(a) I  (b) I 
parallel to one of the sides of the loop and is in the plane c e
of the loop as shown in the figure. Then, the current loop
will (AIIMS 2003) Ic Ie 
(c) I R (d) I R
e c
i1
75. The figure shows three situations when an electron with

i2
velocity v travels through a uniform magnetic field B . In
each case, what is the direction of magnetic force on the
electron ? (AIIMS 2007)

(a) move away from the wire


(b) move towards the wire
(c) remain stationary
(d) rotate about an axis parallel to the wire
72. A circular coil of radius R carries an electric current. The (a) +ve z-axis, –ve x-axis, +ve y-axis
magnetic field due to the coil at a point on the axis of the
(b) –ve z-axis, –ve x-axis and zero
coil located at a distance r from the centre of the coil, such
that r > > R, varies as (AIIMS 2004) (c) +ve z-axis, +ve y-axis and zero
3/2 (d) –ve z-axis, +ve x-axis and zero
(a) 1/r (b) 1/r
2 3 76. A long straight wire of radius a carries a steady current I.
(c) 1/r (d) 1/r
The current is uniformly distributed across its cross-
73. The magnetic field due to a straight conductor of uniform
section. The ratio of the magnetic field at a/2 and 2a is
cross-section of radius a and carrying a steady current is
represented by (AIIMS 2004) (a) 1/4 (b) 4 (AIIMS 2008)
(c) 1 (d) 1/2
77. Statement–1 : The magnetic field produced by a current
(a) (b) carrying solenoid is independent of its length and cross-
sectional area.
Statement–2 : The magnetic field inside the solenoid is
uniform. (AIIMS 2008)
(a) If both Statement–1 and Statement–2 are true and the
(c) (d) Statement–2 is the correct explanation of the Statement–1.
(b) If both Statement–1 and Statement–2 are true but the
Statement–2 is not the correct explanation of the
74. Circular loop of a wire and a long straight wire carry
Statement–1.
currents I c and I e, respectively as shown in figure.
Assuming that these are placed in the same plane. The (c) If Statement–1 is true but Statement–2 is false.
magnetic field will be zero at the centre of the loop when (d) If both Statement–1 and Statement–2 are false.

Mahesh Tutorials Science


MAGNETISM 345

78. If M be the mass of the charged particle, which enters with


velocity v normal to the magnetic field B, it will revolve
with angular speed given by ? (AIIMS 1996)

B q0 B
(a) q M (b)
0 M
q0 M
(c) (d) q0BM
B

Lakshya Educare
346 MAGNETISM

ANSWER KEY
EXERCISE - 1 : BASIC OBJECTIVE QUESTIONS
1. (d) 2. (b) 3. (d) 4. (d) 5. (b) 6. (c) 7. (a) 8. (a) 9. (a) 10. (a)
11. (a) 12. (c) 13. (b) 14. (d) 15. (b) 16. (a) 17. (d) 18. (d) 19. (b) 20. (b)
21. (c) 22. (b) 23. (b) 24. (d) 25. (a) 26. (c) 27. (b) 28. (b) 29. (b) 30. (b)
31. (c) 32. (c) 33. (b) 34. (c) 35. (c) 36. (a) 37. (b) 38. (c) 39. (a) 40. (c)
41. (d) 42. (d) 43. (b) 44. (a) 45. (d) 46. (d) 47. (b) 48. (c) 49. (a) 50. (b)
51. (b) 52. (b) 53. (b) 54. (a) 55. (b) 56. (a) 57. (b) 58. (a) 59. (b) 60. (a)
61. (b) 62. (a) 63. (d) 64. (a) 65. (a) 66. (a) 67. (c) 68. (a) 69. (b) 70. (b)
71. (b) 72. (d) 73. (c) 74. (c) 75. (d) 76. (a) 77. (d) 78. (d) 79. (b) 80. (d)
81. (b) 82. (c) 83. (b) 84. (c) 85. (d) 86. (a) 87. (c) 88. (b) 89. (d) 90. (b)
91. (c) 92. (c) 93. (a) 94. (a) 95. (a) 96. (c) 97. (c)

EXERCISE - 2 : PREVIOUS YEAR COMPETITION QUESTIONS


1. (b) 2. (b) 3. (c) 4. (a) 5. (b) 6. (b) 7. (d) 8. (c) 9. (c) 10. (d)
11. (d) 12. (c) 13. (d) 14. (c) 15. (b) 16. (b) 17. (c) 18. (a) 19. (d) 20. (c)
21. (a) 22. (b) 23. (a) 24. (a) 25. (a) 26. (d) 27. (b) 28. (a) 29. (d) 30. (c)
31. (d) 32. (b) 33. (b) 34. (d) 35. (c) 36. (a) 37. (a) 38. (b) 39. (d) 40. (b)
41. (b) 42. (c) 43. (d) 44. (d) 45. (c) 46. (c) 47. (c) 48. (a) 49. (b) 50. (c)
51. (a) 52. (d) 53. (c) 54. (a) 55. (d) 56. (a) 57. (c) 58. (c) 59. (b) 60. (b)
61. (d) 62. (c) 63. (c) 64. (a) 65. (c) 66. (d) 67. (a) 68. (b) 69. (a) 70. (b)
71. (b) 72. (d) 73. (a) 74. (a) 75. (b) 76. (c) 77. (b) 78. (b)

Dream on !!


Mahesh Tutorials Science

You might also like